Sei sulla pagina 1di 90

Clinical

Pharmacology
&
Therapeutics

Alasdair Scott
BSc (Hons) MBBS PhD

2012
dr.aj.scott@gmail.com
Table of Contents
Pharmacologic Principles ......................................................................................................... 3
Gastrointestinal ...................................................................................................................... 12
Cardiovascular ....................................................................................................................... 18
Respiratory ............................................................................................................................ 33
Central Nervous System ........................................................................................................ 37
Infection ................................................................................................................................. 51
Endocrine ............................................................................................................................... 61
Malignancy and Immunosuppression..................................................................................... 70
Musculoskeletal ..................................................................................................................... 74
Emergencies .......................................................................................................................... 80
Revision ................................................................................................................................. 85
Pharmacologic Principles
Contents
Pharmacokinetics .......................................................................................................................................................................... 4
Drug Metabolism and Elimination.................................................................................................................................................. 4
Adverse Drug Reactions ............................................................................................................................................................... 5
Important ADRs ............................................................................................................................................................................. 6
Side Effect Profiles ........................................................................................................................................................................ 7
Drug Interactions ........................................................................................................................................................................... 8
Prescribing in the Young, the Old and the Pregnant ..................................................................................................................... 9
Prescribing in Renal Disease ...................................................................................................................................................... 10
Prescribing in Liver Disease ........................................................................................................................................................ 11

Alasdair Scott, 2012 3


Pharmacokinetics
Definitions Drug Metabolism and Elimination
Pharmacokinetics: what the body does to the drug
Pharmacodynamics: what the drug does to the body First Pass Metabolism
Metabolism and inactivation of a drug before it reaches
Key Concepts the systemic circulation.
i.e. pre-systemic elimination
Clearance Occurs in gut wall and liver
Volume of plasma cleared of a drug per unit time E.g. propranolol, verapamil, morphine, nitrates

Half-life Pathways of Drug Metabolism and Elimination


Time taken for drug concentration to decline to half its Excrete unchanged by the kidney (e.g. frusemide)
original value. Phase 1 metabolism then renal excretion
Depends on volume of distribution and clearance Phase 2 metabolism then renal excretion

Volume of Distribution Phase 1 Metabolism


Volume into which a drug appears to distribute. Creation of reactive, polar functional groups
High for lipid-soluble drugs Oxidation: usually by CyP450 system
Low for water soluble drugs Reduction and hydrolysis

First Order Kinetics Phase 2 Metabolism


Clearance of drug is always proportional to plasma Production of polar compounds for renal elimination
concentration. Either the drug or its phase 1 metabolite
Most drugs are in this category Conjugation reactions
Glucuronidation, sulfonation, acetylation, methyl
Zero Order Kinetics
Clearance of drug not always proportional to plasma Elimination: mainly renal (depends on GFR)
concentration.
Saturation of metabolism constant rate of elimination Cytochrome P450 System
regardless of plasma levels. Most important system of phase 1 metabolism
E.g. phenytoin, salicylates, ethanol > 11 subtypes
CyP3A4
Bioavailability Most important subtype
Percentage of the dose of a drug which reaches the 30% of drugs: CCBs, -B, statins, benzos
systemic circulation. CyP2D6
100% for IV administration Second most important
20% of drugs: antidepressants, some -B,
opiates
Multiple Dosing
If a drug given at intervals the concentration will reach a Pro-Drugs
steady state in ~ 5 half-lives. L-Dopa dopamine
Loading dose: time needed to reach a steady state. Enalapril enalaprilat
Useful if long or short t Ezetimibe ez-glucuronide
Phenytoin, digoxin, amiodarone, theophylline Methyldopa -methylnorepinephrine
Azathioprine 6-mercaptopurine (by XO)
Carbimazole methimazole
Therapeutic Drug Monitoring Cyclophosphamide

Indications Pharmacogenomics
Ix lack of drug efficacy or possibility of poor compliance Genetically determined variation in drug response
Suspected toxicity
Prevention of toxicity Acetylation
Fast vs. slow acetylators ( fast in Japan vs. Europe)
Examples Affects: isoniazid, hydralazine and dapsone
Aminoglycosides (essential)
Vancomycin (essential) Oxidation
Li (essential) There are genetic polymorphisms for all known CyP
Phenytoin 450 enzymes except for CyP3A4
Carbamazepine
Digoxin G6PD Deficiency
Ciclosporin Oxidative stress haemolysis
Theophylline Quinolones, primaquine, nitrofurantoin, dapsone

NB. Warfarin is not monitored per se, its the biological effect Acute Intermittent Porphyria
which is monitored rather than the plasma drug level. AD, in White South Africans
Alasdair Scott, 2012
Large no. of drugs can attacks: e.g. EtOH, NSAIDs
4
Adverse Drug Reactions
Classification Allergies

Type A Type 1: anaphylaxis


Common, predictable reactions Penicillins, contrast media
Dose-related (but may occur @ therapeutic doses)
Consequence of known pharmacology of the drug Type 2: cytotoxic antibodies
E.g. causing haemolysis
Type B Penicillins, cephalosporins, oral hypoglycaemics
Rare, idiosyncratic reactions Methyldopa
Usually not dose-related
E.g. allergies and pharmacogenetic variations Type 3: immune complexes
Serum sickness-like reaction
Long-Term ADR Penicillins, sulphonamides
Dependence, addiction
Withdrawal phenomena Type 4: cell-mediated
Adaptive changes: e.g. tardive dyskinesia Contact dermatitis
Topical abx
Delayed ADR Antihistamine cream
Cacinogenesis
Teratogenesis
Pseudoallergies
Pharmacological (not immune)
Determinants
NSAIDs bronchospasm
Drug Shift metabolism from prostaglandins leukotrienes
Pharmocodynamics bronchoconstriction
Phamacokinetics May occur in non-asthmatic populations
Dose Commoner if asthmatic
Formulation
Route of administration ACEi cough and angioedema (anaphylactoid)
Rate of aminophylline ACEi inhibit bradykin metabolism

Patient
Age Long-term ADRs
Co-morbidities
Renal: digoxin, aminoglycosides Withdrawal
Hepatic: warfarin, opiates Opiates
Organ dysfunction Benzos
Genetic predisposition Corticosteroids

Rebound: Worse on withdrawing the drug then before starting


Clonidine
-B
Corticosteroids

Adaptive
Neuroleptics: tardive dyskinesia

Delayed ADRs
Oestrogens
Endometrial Ca
Breast Ca

Cytotoxics
Leukaemia

Alasdair Scott, 2012 5


Important ADRs
Rashes Peripheral Neuropathy
INH
Urticaria: Vincristine
Immune: penicillins, cephalosporins Amiodarone
Non-immune: contrast, opiates, NSAIDs Nitrofurantoin
Penicillamine
EM: sufonamides NSAIDs, allopurinol, phenytoin, penicillin

EN: sulphonamides, OCP


Pulmonary Fibrosis
Photosensitivity: amiodarone, thiazides, sulfonylureas Bleomycin
Busulfan
Fixed eruptions: erythromycin, sulphonamides Amiodarone
Nitrofurantoin
Lupus-like reactions: hydralazine, isoniazid, penicillamine Sulfasalazine
Methotrexate
Methysergide
Hepatotoxicity
Cholestatic Gynaecomastia
Clavulanic acid: may be delayed Spironolactone
Fluclox: may be delayed Digoxin
Erythromycin Verapamil
Sulfonylureas (glibenclamide) Cimetidine
OCP Metronidazole
Tricyclics
Chlorpromazine, prochlorperazine
SIADH
Hepatocellular Damage
Carbemezapine
Paracetamol
Cyclophosphamide
Valproate, phenytoin, CBZ
Chlorpropamide
RMP, INH, PZA
SSRIs
Halothane
TCAs
Methotrexate
Statins

Chronic Hepatitis Gingival Hypertrophy


INH Nifedipine
Methyldopa Phenytoin
Methotrexate Ciclosporin

Gallstones
OCP QTc
Fluoroquinolones: cipro
Venlafaxine
Neuroleptics: phenothiazines, haldol
Bone Marrow Toxicity
Macrolides
Pancytopenia / aplastic anaemia Anti-arrhythmics 1a/III: quinidine, amiodarone, sotalol
Cytotoxics TCAs
Phenytoin Histamine antagonists
Chloramphenicol
Penicillamine
Phenothiazines
Methyldopa

Neutropenia
Carbamazapine
Carbimazole
Clozapine
Sulfasalazine

Thrombocytopenia
Valproate
Salicylates
Chloroquine
Alasdair Scott, 2012 6
Side Effect Profiles
Cholinoceptors EPSEs
Cholinergic Anti-muscarinic Causes
Salivation Constipation Typical antipsychotics
Bronchoconstriction Urinary retention Rarely: metoclopramide, prochlorperazine
Lacrimation Mydriasis Esp. in young women
Urination Blurred vision Dyskinesias and dystonias are common c anti-
Diarrhoea Bronchodilatation parkinsonian drugs.
GI upset Drowsiness
Emesis Dry eyes / skin Mechanism
Miosis D2 block in the nigrostriatal pathway
Sweating Excess AChM (hence effect of anti-AChM)

Causes Causes
Anti-cholinesterases Ipratropium Parkinsonian
Anti-histamines Occurs w/i months
TCAs Commoner in the elderly
Antipsychotics Bradykinesia tremor, rigidity
Procyclidine Rx: procyclidine (anti-AChM)
Atropine
Acute Dystonia
Occurs w/i hrs-days of starting drugs
Dopamine Commoner in young males
Involuntary sustained muscle spasm
Excess E.g. lock jaw, spasmodic torticollis, oculogyric crisis
Causes Rx: procyclidine
L-dopa
Da agonists
Features Akathisia
Behaviour change Occurs w/i days-months
Confusion
Subjective feeling of inner restlessness
Psychosis
Rx: propranolol (crosses BBB)
Deficit
Causes
Tardive Dyskinesia
Anti-psychotics
Rhythmic involuntary movements of head, limbs and
Anti-emetics: metoclopramide, prochlorperazine
trunk.
Features
Chewing, grimacing
EPSEs
PRL Protruding, darting tongue
Neuroleptic malignant syndrome Occur in 20% of those on long-term neuroleptics (yrs)
Rx
Switch atypical neuroleptic
Clozapine may help
Cerebellum
(procyclidine worsens symptoms)
Dysdiadochokinesis, dysmetria, rebound
Ataxia
Nystagmus Neuroleptic Malignant Syndrome
Intention tremor 4-10d after initiation or change of dose
Slurred speech Mostly in young males
Hypotonia Features
Motor: severe muscular rigidity
Mental: fluctuating consciousness
Causes Autonomic: hyperthermia, HR, sweating, /BP
EtOH Blood: CK, leukocytosis
Phenytoin Rx
Dantrolene: inhibits muscle Ca release
Bromocriptine / apomorphine: reverse Da block
Cool pt.

Alasdair Scott, 2012 7


Drug Interactions
Pharmaceutical P450 Inducers
Take place outside the body Phenytoin
Mainly IV drugs being mixed together Carbamazepine
E.g. calcium + sodium bicarbonate precipitation Barbiturates
Rifampicin
Alcohol (chronic)
Pharmacokinetic Griseofulvin
St. Johns Wart
Altered Absorption
Tetracyclines and quinolones
c Ca, Fe, Al
Drugs chelate the metals and are not absorbed P450 Inhibitors
Valproate
INH
Displacement from Plasma Proteins Protease inhibitors
Warfarin + some NSAIDs Cipro, Cimetidine
Often clinically insignificant as clearance s Erythromycin + clarithromycin
proportionally
c displacement Omeprazole
Grapefruit juice
Fluconazole / Fluoxetine
Metabolism
Enzyme Inhibitors
P450 Important Drugs Metabolised by P450
XO: allopurinol Ciclosporin
Dopa decarboxylase: carbidopa OCP
Acetaldehyde dehydrogenase: disulfiram,
Warfarin
metronidazole
Epileptic drugs: phenytoin, CBZ
Enzyme Inducers
Statins
Theophylline
Excretion
Diuretics Li clearance
Loop diuretics: aminoglycoside ototoxicity
Warfarin
W+ W-
Indirect Interactions Enzyme inhibitors Enzyme inducers
Diuretics and steroids risk of digoxin toxicity EtOH
Via K+ Simvastatin
NSAIDs + warfarin risk of GI bleed NSAIDs
Abx + warfarin bleeding risk Dipyridamole
Abx kill GI microflora that make vit K Amiodarone

Diuretics
Potentiate: ACEi, Li, Digoxin
Loop risk of ototoxicity
c aminoglycosides
K-sparing risk of hyperkalaemia c ACEi

Alasdair Scott, 2012 8


Prescribing in the Young, the Old and the Pregnant
The Elderly Teratogens
Pharmacokinetics Mechanisms
Distribution Orally active = crosses placenta
body water [water soluble drugs] Implantation (5-17d) abortion
body fat [fat soluble drugs] Embryonic (17-57d) structural defects
albumin [protein-bound drugs] Fetogenic (maturation) relatively less dangerous
wt. standard dose [drug]
Metabolism Common Drugs
oxidation ACEi: affect kidney growth
first-pass metabolism (e.g. propranolol) AEDs: NTDs
induction of liver enzymes Li: Ebsteins anomaly
age t of hepatically metabolised drugs Anti-folate: e.g. trimethoprim NTDs
E.g. Warfarin Tetracyclines: stain teeth
Elimination Warfarin: cardiac defects, IQ, saddle nose, blindness
GFR Statins
tubular secretion

Bottom Line
age tends to greater and longer drug effects
Drugs to Avoid in Late Pregnancy
Aspirin: haemorrhage, kernicterus
Altered Organ Sensitivity
Aminoglycosides: CN8 damage
ANS
Anti-thyroid: goitre, hypothyroidism
Defective compensatory mechanisms
-receptor density Benzos: floppy baby syndrome
effectiveness of drugs targeting them Chloramphenicol: grey baby syndrome
CNS Warfarin: haemorrhage
sensitivity to anxiolytics and hypnotics Sulphonylureas: kernicterus
Cardiac Function
perfusion of liver and kidneys function
metabolism or elimination of drug
Mx in Pregnancy
Compliance Problems
Confusion changes in tablet morphology Hypertension
vision NB. Dont prescribe ACEi to fertile young women.
Arthritic hands Labetalol
Living alone Methyldopa
Polypharmacy Nifedipine
Hydralazine
Major Problem Drugs
Affecting the Cardiovascular System DM
Anti-hypertensives Poor glucose control congenital abnormalities
Digoxin Use insulin and/or metformin
Diuretics
Affecting the CNS Epilepsy
Anti-depressants Folic acid pre-conception
Ant-parkinsonian Drug levels tend to fall in pregnancy
Hypnotics risk of malformations (6% vs. 2%)
risk of haemorrhagic disease of the newborn (K)
Neonates Avoid valproate
Use lamotrigine (or CBZ)
Pharmacokinetics
Absorption: gastric motility Anticoagulation
Distribution st
1 trimester: LMWH
Immature BBB 2nd trimester 36wks: LMWH or warfarin
body water [water soluble drugs] 36wks term: LMWH
body fat [fat soluble drugs]
Term onwards: warfarin
albumin [protein-bound drugs]
Metabolism
P450 activity
conjugation Drugs and Breast-Feeding
Excretion Sedatives (benzos, EtOH) drowsiness
GFR and tubular secretion Anti-thyroid goitre
Tolbutamide hypoglycaemia in infant
Bottom Line
age tends to greater and longer drug effects
Alasdair Scott, 2012 9
Prescribing in Renal Disease
Important Drugs Affected by Renal Impairment Nephrotoxicity
Digoxin Gentamicin
T: 36 90hrs Renal tubular damage
Low therapeutic index: should be monitored Accumulation nephro-/oto-toxicity
Toxicity Must monitor drug levels
Nausea, xanthopsia, gynaecomastia
A / V tachyarrhythmias, heart block Li
Inhibits Mg-dependant enzymes
Gentamicin e.g. adenylate cyslase
T: 2.5 >50hrs ADH requires adenylate cyclase Li
Must be monitored nephrogenic DI
Toxicity Also causes direct tubular damage
Ototoxic: hearing + vestibular Must monitor drug levels
Nephrotoxic tubules
risk of toxicity if Na (e.g. diuretics) or dehydration Ciclosporin
GFR: reversible
Atenolol Damages renal tubules: irreversible
T: 6 100hrs P450 substrate
Toxicity Consider monitoring
Bradycardia, hypotension
Worsening of PVD and heart failure ACEi / ARB
Confusion GFR: inhibit efferent arteriorlar vasoconstriction
CI May be profound in RAS or CoA
Asthma / bronchospasm
Severe heart failure NSAIDs
PVD GFR: prevent afferent arteriolar vasodilatation
Papillary necrosis
Amoxicillin
T: 2 14hrs
Toxicity
Seizures (in meningitis: impaired BBB)
Rashes

Captopril
T: 2 14hrs
Toxicity
Hypotension
Taste disturbance
Cough
GFR
Angioedema

Vitamin D
Forms
Colecalciferol / D3: formed in skin and found in food
Ergocalciferol / D2: produced by UV light in fungi from
ergosterol
Calcifediol: 25 OH-Vit D3 produced by the liver
Calcitriol: 1, 25, (OH)2 Vit D3, produced by the kidney
Alfacalcidol: 1 OH-Vit D3

Pathophysiology
25 OH-Vit D3 undergoes renal 1-hydroxylation to the
active form: 1, 25, (OH)2 Vit D3
Impaired 1-hydroxylase function in renal disease

In Renal Impairment
Use alfacalcidol (1-hydroxylated)
Calcitriol rarely used

Alasdair Scott, 2012 10


Prescribing in Liver Disease
Cautions in Hepatic Impairment Hepatotoxicity
Hypoalbuminaemia Cholestatic
proportion of free drug Clavulanic acid: may be delayed
Phenytoin, CBZ Fluclox: may be delayed
Prednisolone Erythromycin
Diazepam Sulfonylureas (glibenclamide)
Tolbutamide OCP
Tricyclics
Synthesis of Clotting Factors Chlorpromazine, prochlorperazine
Warfarin
Abx (kill GI microflora) Hepatocellular Damage
Paracetamol
First Pass Metabolism Valproate, phenytoin, CBZ
Opiates RMP, INH, PZA
Phenothiazines Halothane
Imipramine Methotrexate
Statins
1-acidic Glycoprotein
Binds basic drugs Chronic Hepatitis
e.g. chlorpromazine, quinidine, imipramine INH
Methyldopa
Encephalopathy
Sedatives / Opiates: may coma Gallstones
Caution
c drugs that may constipation OCP
Anxiolytics: temazepam safest (short t)
Avoid chlormethiazole
Anti-depressants: TCAs safest (but dose)
Avoid MOAIs

Hepatorenal Syndrome
Withdraw nephrotoxic drugs
Modify doses of renally-excreted drugs

Alasdair Scott, 2012 11


Gastrointestinal
Contents
Dyspepsia, GORD and PUD ....................................................................................................................................................... 13
Laxatives ..................................................................................................................................................................................... 14
Miscellaneous Gastrointestinal Drugs ......................................................................................................................................... 14
GORD .......................................................................................................................................................................................... 15
PUD ............................................................................................................................................................................................. 15
H. pylori Eradication Therapy ...................................................................................................................................................... 15
Ulcerative Colitis .......................................................................................................................................................................... 16
Crohns Disease .......................................................................................................................................................................... 16
C. diff Diarrhoea .......................................................................................................................................................................... 17

Alasdair Scott, 2012 12


Dyspepsia, GORD and PUD

Drug MOA Side Effects Contraindications Interactions Other


Mg Trisilicate Ant-acid Diarrhoea Interfere
c drug absorption Take when symptoms occur or are
Neutralise gastric acid - take separately expected.
Al hydroxide Ant-acid Constipation Interfere
c drug absorption Take when symptoms occur or are
Neutralise gastric acid - take separately expected.
Gaviscon Alginate

reflux
- viscosity of stomach contents
- Form a raft on top of stomach
contents
Omeprazole PPIs GI disturbance P450 inhibitor Caution
Lansoprazole Headache - may mask symptoms of
Pantoprazole Activated in acidic pH gastric Ca
Irreversibly inhibit H+/K+ ATPase
More effective cf. H2RAs
Cimetidine H2 receptor antagonist Mainly c cimetidine Cimetidine is a P450 inhibitor Caution
Ranitidine GI disturbance - may mask symptoms of
gastric parietal cell H+ secretion gastric Ca
Misoprostol Prostaglandin analogue Diarrhoea is v. common Mainly used to prevent NSAID-assoc.
PUD.
Acts on parietal cells to H+ secretion
Often in combination
c NSAID
- Diclofenac + misoprostol = Arthrotec

Alasdair Scott, 2012 13


Laxatives

Drug MOA Side Effects Contraindications Interactions Other


Bran Bulk laxatives Bloating Bowel obstruction
Ispaghula faecal mass peristalsis
Docusate Stimulant laxative Bowel obstruction Co-danthrusate is a mild
Glycerin (PR) intestinal motility stimulant laxative used in
Senna Rx of opioid-induced
Picosulfate constipation.
Lactulose Osmotic laxative Bowel obstruction Lactulose is used in Rx of
Macrogol stool water content hepatic encephalopathy
Phosphates (PR)
Mg Salts
Liquid paraffin Stool softener absorption of ADEK vitamins Bowel obstruction
Granulomatous reactions

Miscellaneous Gastrointestinal Drugs

Drug MOA Side Effects Contraindications Interactions Other


Hyoscine butylbromide Antimuscarinic Anti-AChM SEs Myasthenia gravis
- Buscopan - Antispasmodic - Dry mouth
- Palpitations
Mebeverine Antispasmodic
Peppermint oil
Loperamide Opioid receptor agonist Abdo cramps Infective diarrhoea
Doesnt cross BBB no central Colitis
effects. Caution in hepatic
impairment
Sulfasalazine 5-Aminosalicylate Sulfasalazine has SEs Caution in renal or Monitor FBC
Mesalazine - blood dyscrasias hepatic impairment
Unknown MOA - hepatitis Topical in distal disease
- rash, urticaria
- oligospermia
- pulmonary fibrosis
Budesonide Steroid Used to induce remission in
More potent cf. prednisolone ileal Crohns
High 1st pass metabolism
systemic effects.
Infliximab: Remicade Chimeric anti-TNF mAb Severe infections TB Screen for TB before use
Etanercept: Enbrel P75 TNFR-Fc fusion protein TB Parenteral admin
Adalimumab: Humira Human anti-TNF mAb Allergic reactions
CCF Give c hydrocortisone to
CNS demyelination allergic SEs
Alasdair Scott, 2012 14
GORD PUD
Conservative Conservative
Lose wt. Lose wt.
Raise head of bed Stop smoking and EtOH
Small regular meals 3h before bed Avoid hot drinks and spicy food
Stop smoking and EtOH Stop drugs: NSAIDs, steroids
Avoid hot drinks and spicy food
Avoid tight clothes Medical
Stop drugs: NSAIDs, steroids, CCBs, nitrates OTC antacids: Gaviscon, Mg trisilicate
H. pylori eradication: PAC500 or PMC250
Medical Full-dose acid suppression for 1-2mo
OTC antacids: Gaviscon, Mg trisilicate PPIs: lansoprazole 30mg mane
1: Full-dose PPI for 1-2mo H2RAs: ranitidine 300mg nocte
Lansoprazole 30mg OD Low-dose acid suppression PRN
2: No response double dose PPI BD
3: No response: add an H2RA Surgical
Ranitidine 300mg nocte Rarely performed
Control: low-dose acid suppression PRN Selective vagotomy
Antrectomy + vagotomy
Surgical: Nissen Fundoplication Subtotal gastrectomy + Roux-en-Y
Indications: all 3 of:
Severe symptoms
Refractory to medical therapy
Confirmed reflux (pH monitoring) H. pylori Eradication Therapy
7 days Rx
NB. PPIs and cimetidine false ve C13 breath tests
and antigen tests stop >2wks before.

PAC 500
PPI: lansoprazole 30mg BD
Amoxicillin 1g BD
Clarithromycin 500mg BD

PMC 250
PPI: lansoprazole 30mg BD
Metronidazole 400mg BD
Clarithromycin 250mg BD

Failure
95% success
Mostly due to poor compliance
Add bismuth
Stools become tarry black

Alasdair Scott, 2012 15


Ulcerative Colitis Crohns Disease
Acute Severe UC Acute Severe Crohns
Resus: Admit, IV hydration, NBM
Hydrocortisone: IV 100mg QDS + PR Assessment
Transfuse if required temp, HR, ESR, CRP, WCC, albumin
Thromboprophylaxis: LMWH
Monitoring Management
Bloods: FBC, ESR, CRP, U+E Resus: Admit, NBM, IV hydration
Vitals + stool chart Hydrocortisone: IV + PR if rectal disease
Twice daily examination Abx: metronidazole PO or IV
AXR Thromboprophylaxis: LMWH
Dietician Review
Improvement oral therapy Elemental diet
Switch to oral pred + 5-ASA Consider parenteral nutrition
Taper pred after full remission Monitoring
Vitals + stool chart
No Improvement rescue therapy Daily examination
Discussion between pt, physician and surgeon
Medical: ciclosporin, infliximab or visilizumab Improvement oral therapy
Surgical Switch to oral pred (40mg/d)

No Improvement rescue therapy


Discussion between pt, physician and surgeon
Inducing Remission in Mild to Moderate Disease Medical: methotrexate infliximab
OPD treatment Surgical
Oral Therapy
1st line: 5-ASAs Inducing Remission in Mild to Moderate Disease
2nd line: prednisolone OPD treatment
3rd line: ciclosporin or infliximab
Supportive
Topical Therapy: mainly left-sided disease High fibre diet
Proctitis: suppositories Vitamin supplements
More proximal disease: enemas or foams
5-ASAs steroids (prednisolone or budesonide) Oral Therapy
1st line
Additional Therapy: steroid sparing Ileocaecal: budesonide
Azathioprine Colitis: sulfasalazine
Infliximab: steroid-dependent pts 2nd line: prednisolone (tapering)
3rd line: methotrexate
4th line: infliximab or adalimumab
Maintaining Remission
Perianal Disease
1st line: 5-ASAs PO sulfasalazine or mesalazine
Occurs in ~50%
Topical Rx may be used in proctitis
Ix: MRI + EUA
2nd line: azathioprine or 6-mercaptopurine
Relapsed on ASA or are steroid-dependent Rx
Use 6-mercaptopurine if azathioprine intolerant Oral Abx: metronidazole
Immunosuppression infliximab
3rd line: infliximab / adalimumab
Local surgery seton insertion

Maintaining Remission
Elective Surgery 1st line: azathioprine or mercaptopurine
2nd line: methotrexate
Indications 3rd line: Infliximab / adalimumab
Chronic symptoms despite medical therapy
Carcinoma or high-grade dysplasia
Elective Surgery
Procedures
Indications
Proctocolectomy c end ileostomy or IPAA
Abscess or fistula
Total colectomy
c (IRA)
Perianal disease
Chronic ill health
Carcinoma

Procedures
Limited resection: e.g. ileocaecal
Stricturoplasty
Alasdair Scott, 2012
Defunction distal disease c temporary loop ileostomy
16
C. diff Diarrhoea
High Risk Abx
Cephalosporins
Clindamycin

General
Stop causative Abx
Avoid antidiarrhoeals and opiates
Enteric precautions

Specific
1st line: Metronidazole 400mg TDS PO x 10-14d

Metronidazole Failure
Vanc 125mg QDS PO x 10-14d

Severe: Vanc 125mg QDS PO 1st line (may add metro IV)
to 250mg QDS if no response (max 500mg)
Urgent colectomy may be needed if
Toxic megacolon
LDH
Deteriorating condition

Recurrence (15-30%)
Reinfection or residual spores
Repeat course of metro x 10-14d
Vanc if further relapse (25%)

Treatment Failure
Defined as no clinical response after 1wk
C. diff toxin assay will remain positive for 2wks
following original infection.

Alasdair Scott, 2012 17


Cardiovascular
Contents
Diuretics ....................................................................................................................................................................................... 19
The Renin-Angiotensin System ................................................................................................................................................... 20
Beta Blockers .............................................................................................................................................................................. 21
Miscellaneous Anti-Hypertensives .............................................................................................................................................. 22
Calcium Channel Blockers .......................................................................................................................................................... 23
Nitrates ........................................................................................................................................................................................ 24
Miscellaneous Anti-Anginals ....................................................................................................................................................... 24
Vaughan-Williams Classification ................................................................................................................................................. 25
Anti-Arrhythmic Agents ................................................................................................................................................................ 26
Anti-Platelet Agents ..................................................................................................................................................................... 27
Lipid Lowering Therapy ............................................................................................................................................................... 28
Warfarin ....................................................................................................................................................................................... 29
Heparin ........................................................................................................................................................................................ 30
Thrombolytics .............................................................................................................................................................................. 30
Atrial Fibrillation ........................................................................................................................................................................... 31
Angina Pectoris ........................................................................................................................................................................... 31
Chronic Heart Failure .................................................................................................................................................................. 32
Hypertension ............................................................................................................................................................................... 32

Alasdair Scott, 2012 18


Diuretics

Drug MOA Side Effects Contraindications Interactions Other


Frusemide Loop diuretics Na Refractory hypokalaemia toxicity of Monitor U+Es
Bumetanide K Anuric renal failure - digoxin (due to K)
Inhibit Na / K / Cl triple transporter in Ca - NSAIDs May add K+-sparing diuretic to
ascending loop of henle Mg - Gentamicin K loss.
- NaCl excretion urate - Li
Postural hypotension
Tinnitus / deafness (rare)
Bendrofluazide Thiazide diuretics Na Refractory hypokalaemia toxicity of
Metolazone K Gout - digoxin
Chlortalidone Inhibit NaCl transporter in DCT Ca Severe renal failure - Li
- NaCl excretion urate
Postural hypotension
glucose and DM
Spironolactone Aldosterone receptor antagonists K K toxicity of Spiro doses
Eplerenone - Na excretion Gynaecomastia (spiro) P - digoxin - 25mg OD for HF
- K and H excretion Addisons - Li - 100-400mg OD for diuresis
Amiloride Block Na channels in collecting K c K+-
Typically used in combo
Triamterine tubules GI upset wasting diuretics.
- Na excretion
- K and H excretion
Acetazolamide Carbonic anhydrase inhibitor Rash: EM SJS Sulfonamide hypersensitivity A sulphonamide
- HCO3 excretion Peripheral tingling
Use
- open- / closed-angle glaucoma
Mannitol Osmotic diuretic Use
- IOP
- ICP

Alasdair Scott, 2012 19


The Renin-Angiotensin System

Drug MOA Side Effects Contraindications Interactions Other


Lisinopril ACEi Hypotension Suspected or confirmed bilat risk of RF
c NSAIDs Monitor U+Es
Captopril - esp.
c diuretics, HF RAS - Cr >30% MRA
Ramipril Inhibits conversion of ang I ang II RF Diuretics, TCAs and
Perindopril K+ Angioedema/hypersensitivity antipsychotics risk of BP Titrate dose
Dry cough: 10-20% to ACEi
- 2O to bradykinin Caution c drugs that K+ Avoid in young women who
+
Angioedema (~0.1%) Salt substitutes (contain K ) - e.g. spiro might become pregnant
- consider -B
P/B
dose in RF
Candesartan Ang II receptor (AT1) antagonists As for ACEI but no cough P/B As above
Irbesartan
Losartan Dont inhibit kinin breakdown no Caution in RAS
cough

Physiology Principal Indications


Angiotensinogen is an 2-globulin released by the liver. Heart failure
Renin from the JGA converts angiotensinogen ang 1 HTN
ACE is produced by pulmonary epithelial cells and converts ang 1 ang 2. Post-MI
Ang 2 acts via AT1 receptor Angina
Vasoconstriction Diabetic Nephropathy
Sympathetic activation
Aldosterone release from adrenal cortex
renal Na reabsorption
ADH release
Ang 2 is degraded by angiotensinases in RBCs

Alasdair Scott, 2012 20


Beta Blockers

Drug MOA Side Effects Contraindications Interactions Other


Cardioselective Block receptors Bronchospasm Asthma / bronchospasm Verapamil and Propranolol is v. lipid
- Bisoprolol - inc. cardioselective PVD diltiazem risk of AV soluble and easily
- Atenolol Actions via 1 CO Severe bradycardia block and HR crosses the BBB CNS
- Metoprolol - heart rate HR and BP Severe heart failure effects (nightmares).
- Esmolol - contractility 2nd-3rd degree AV block Enhanced BP effects
- Nebivolol - small BP: central effect + renin Peripheral vasoconstriction c other anti-HTN drugs
Atenolol is water soluble
- cold extremities and doesnt cross BBB.
Non-selective Effects - worsened Raynauds/PVD Caution in DM as likelihood of Blocks symptoms of
- Carvedilol - diastolic perfusion hypoglycaemia glucose
c insulin
- Propranolol - O2 demand Lethargy / fatigue - mask symptoms
- Sotalol - afterload Nightmares
- Labetalol dose in renal impairment
Metabolic May dose in hepatic impairment
ISA - HDL
- Acebutolol - TGs
- Pindolol - risk of new onset DM (esp.
- Oxprenolol c thiazides)

Vasodilating
- Carvedilol
- Labetalol
- Nebivolol

Physiology Pharmacology Principal Indications


Angina
1 receptors: Some -B have arteriolar dilating effects which TPR Heart failure
Heart: rate and contractility Block 1 receptors Acute MI
Kidney: renin release from JGA Carvedilol, labetalol, nebivolol Arrhythmias
HTN
2 receptors: Cardioselective agents have 2 effects. Long QT syndrome
Bronchi, GIT: SM relaxation Prophylaxis vs. variceal haemorrhage
Skeletal muscle: arteriolar dilatation ISA: partial agonist activity at adrenoceptors Migraine prophylaxis
Liver + skeletal muscle: glycogenolysis and bradycardia Thyrotoxicosis
gluconeogenesis cold extremities Glaucoma
Anxiety
3 receptors: Lipophilic compounds more likely to CNS effects
Adipose tissue: lipolysis Propranolol, metoprolol

Hydrophilic compounds may accumulate in renal


failure
Atenolol, sotalol

Alasdair Scott, 2012 Esmolol is v. short acting: use IV 21


Miscellaneous Anti-Hypertensives

Drug MOA Side Effects Contraindications Interactions Other


Non-Selective -blockers Postural Hypotension B s hypotensive effects of Phentolamine is short-
- Phenoxybenzamine Dizziness - diuretics acting and can be used to
- Phentolamine 1 Headache - -B control BP in phaeo.
- systemic vasodilatation Urinary incontinence - CCBs
Alpha 1 - relaxation of internal urethral - esp. women Phenoxybenzamine is
- Doxazosin sphincter Blurred vision long-acting and is used to
- Prazosin maintain blockade once
BP controlled.

Doxazosin and tamsulosin


are used in Rx of BPH
Clonidine Centrally-acting 2 agonist Rebound HTN on withdrawal
- CO Postural hypotension
- PVR Constipation
Nausea
Dry mouth
Methyldopa Centrally-acting 2 agonist Blood dyscrasias L Avoid w/i 2wks of MAOI Mainly used in P
Pro-drug Hepatotoxic Depression
- -methyl NA Drug-induced lupus
Drowsiness
Hydralazine Vasodilator Drug-induced lupus SLE Mainly used in P
- arteries > veins HR
GI upset dose in hepatic or renal
Headache impairment
Nitroprusside Vasodilator Hypertensive crisis
- arteries > veins
Minoxidil Vasodilator Hypertrichosis Used to promote hair
growth

Alasdair Scott, 2012 22


Calcium Channel Blockers

Drug MOA Side Effects Contraindications Interactions Other


Dihydropyridines Mainly Arterial SM Activity Flushing Cardiogenic shock Risk of BP
c /-B Indications
- Nifedipine - vasodilatation (inc. coronary) Headache - Angina
- Amlodipine - particularly pre-capillary arterioles Ankle oedema Unstable angina Fx d by grapefruit - Prinzmetals angina
- esp. amlodipine - HTN
TPR sympathetic tone HR Dizziness Significant AS Fx d by: - Raynauds
BP - Rifampicin
w/i 1mo of MI - CBZ + phenytoin
Gingival hypertrophy
- esp. nifedipine Nifedipine only
- s fx of digoxin

Non-dihydropyridines Mainly Cardiac Activity Headache HF Risk of AV block, HF and Indications


- Diltiazem - -ve inotropic effect (esp. verapamil) Flushing 2nd/3rd degree AV block asystole
c -blockers. - Angina
- Verapamil - verapamil also slows conduction @ AV block - HTN
SA and AV nodes HF s fx of digoxin - Arrhythmias (Verap)
BP
Some activity @ arterial SM (<DHPs) Ankle oedema Fx of verapamil d by:
Constipation - Grapefruit juice
Gynaecomastia (verap) - Macrolides

risk of myopathy
c
simvastatin

Pharmacology Indications
MOA Verapamil and Diltiazem
Bind 1 subunit of L-type Ca channel at distinct sites HTN
Prevent channel opening and inhibit Ca entry Angina
AF
Effects
All CCBs are vasodilators: afterload Nifedipine MR and Amlodipine
Also dilate coronary arteries HTN (long-acting)
Pre-capillary vasodilatation transudative oedema Angina: esp. good for Prinzmetals
Dihydropyridines act only @ arterial SM and can reflex tachycardia Raynauds
Avoid short acting preparations
Verapamil is highly negatively inotropic
CI in HF and c -B
Verapamil is also negatively chronotropic
Diltiazem is less negatively inotropic and chronotropic than verapamil

Alasdair Scott, 2012 23


Nitrates

Drug MOA Side Effects Contraindications Interactions Other


GTN NO donor c rapid onset and short BP (inc. postural) AS and MS Sildenafil, tadakafil and SL spray or tabs
duration (30min) Headache BP vardenafil are CI BP (300ug)
Syncope Constrictive pericarditis
High 1st pass metabolism Dizziness Tamponade s fx of heparins if given IV Use for relief of pain
Flushing HOCM in angina, ACS
Mainly venodilation preload Reflex tachycardia Hb
Small coronary vasodilatation Glaucoma (closed)
ISMN / ISDN Long-acting nitrates Hypovolaemia
ICP
ISMN is active metabolite of ISDN

MN avoids unpredictable first-pass


metabolism of DN

Tolerance develops quickly: need 8h


drug-free period (usually @ night)

Miscellaneous Anti-Anginals

Drug MOA Side Effects Contraindications Interactions Other


Nicorandil K+ATP channel activator + nitrate Headache Cardiogenic shock Sildenafil BP Indications
component Flushing - Uncontrolled angina
Arterial and venous dilator Dizziness
GI ulcers (rare)
Ivabradine Inhibitsfunny current in SA node Visual changes BP or HR Subject to hepatic induction / inhibition Indication
pacemaker activity HR HR and HB ACS - Angina (useful if -B CI)
Strong CYP inhibitor
Trimetazidine Inhibits fatty acid oxidation Indication
myocardial glucose use - Angina
Ranolazine Inhibits late Na current Indication
- Angina

Alasdair Scott, 2012 24


Vaughan-Williams Classification
Class I Class IV
Na+ channel blockers (local anaesthetics). CCBs: verapamil and diltiazem (non-DHPs)
All slow conduction Site of action: AVN
Subclasses have additional effects on action potential MOA: Slow AVN conduction
Use-dependent: preferentially block open or refractory Na+ channels Indications: prevent recurrence of SVT, AF (rate control), acute SVT
Caution: negative inotropes
1a: quinidine, procainamide, disopyramide
Sites of action: A, SAN, AVN, V Unclassified
Repolarisation: Prolonged AP duration
Indications: V arrhythmias Digoxin
SEs: anti-AChM, negative inotropes Cardiac glycoside
MOA
1b: lignocaine, mexiletine Positive inotrope: inhibits myocyte Na+/K+ ATPase Na+ & Ca2+
Sites of action: V only Negative chronotrope
Repolarisation: Shortened normal or AP duration Slows AV conduction rate and AVN refractory period
Indications: V arrhythmia following MI Only slows resting rate, NOT exercise rate
Indication: SVT and AF
1c: flecainide Caution
Sites of action: A, SAN, AVN, V Toxicity can arrhythmias
Repolarisation: Little effect toxicity if K (reduced competition for binding site)
Indications: Pre-excited AF (WPW), Acute AF
Adenosine
MOA
Class II Acts @ A1 receptors in cardiac tissue myocyte hyperpolarization
-Blockers: metoprolol, propranolol, esmolol, atenolol Transient AV block
Sites of action: A, SAN, AVN, V Indication: SVTs
MOA
refractory period of AVN slow AVN conduction
Prevent arrhythmias due to sympathetic discharge (e.g. following MI)
Indications: post-MI, AF (rate control), SVT Clinical Classification
Caution: negative inotropes
AVN
Adenosine
Class III -B
Amiodarone and sotalol Verapamil
Sites of action: A, SAN, AVN, V Digoxin
MOA Use: AF, SVT
K+ channel blockers
refractory period = QTc Atria and Ventricles
Indications: V and SV arrhythmias, pre-excited AF (WPW) Class 1a, 1c and amiodarone
Caution: can arrhythmias (esp. TDP)
Ventricles only
Class 1b

Alasdair Scott, 2012 25


Anti-Arrhythmic Agents

Drug Class Indications Side Effects Contraindications Interactions Other


Disopyramide Class Ia Ventricular arrhythmias VT, VF, TdP Heart block (2/3)
(esp. post-MI) Anti-muscarinic
Lignocaine Class Ib Ventricular arrhythmias Drowsiness Heart block Cimetidine s lidocaine levels IV use only
(esp. post-MI) Paraesthesia
Dizziness
Bradycardia cardiac arrest
Flecainide Class Ic Pre-excited AF (WPW) Strong ve inotrope Structural heart
Cardioversion in AF - Oedema disease
Suppress V ectopics - Dyspnoea Post-MI
Amiodarone Class III SVT Eye corneal microdeposits Thyroid disease -B and CCB risk of HB Accumulates in body:
AF / flutter Sinus bradycardia - V. long t (10-100 days)
Pre-excited AF Thyroid hyper / hypo QTc TdP s levels of - Extensively tissue bound
V arrhythmias (inc. VF) - digoxin (halve dig dose) requires loading dose
Lung pulmonary fibrosis - warfarin
- phenytoin Monitor:
GI/Liver LFTs - TFTs, LFTs (base + 6-moly)
N/V risk of V arrhythmias c: - K+ (baseline)
- Class III/Ia antiarrhythmics - CXR (baseline)
Neuro peripheral neuropathy - TCAs, antipsychotics
- Erythromycin Avoid sunlight
Skin photosensitivity
blue-grey discoloration
phlebitis (give centrally)
Digoxin Cardiac AF / flutter Toxicity Complete heart block Dig fx/toxicity d by: Caution
glycoside SVT - Any arrhythmia VF/VT - CCB (esp. verapamil) - Renal excretion caution
(HF) - e.g. SVT c AV block HOCM - Amiodarone (halve dig dose) in impairment
- Nausea SVTs 2O to WPW - Diuretics (loop/thiazide due - e.g. in the elderly
- Xanthopsia to K+)
- Confusion Monitor
- K+ digoxin intestinal absorption: - U+Es
- Antacids - Drug levels (6h post-dose)
Chronic - Cholestyramine
- Gynaecomastia Load then maintenance

Reverse tick ECG


- not a sign of toxicity
Adenosine SVT: Dx and Rx Bronchospasm Asthma Fx prolonged by dipyridamole t = 8-10secs
Chest pain Heart Block (2/3)
Flushing Sick sinus syndrome Fx by theophylline
Nausea
Light-headedness

Alasdair Scott, 2012 26


Anti-Platelet Agents
Drug MOA Side Effects Contraindications Interactions Other
Aspirin Irreversible, non-selective COX inhibitor Gastritis <16yrs (Reyes syn.) risk of bleeding
c other Stop 7d before surgery if
plat TxA2 Gastric ulceration - except in Kawasakis anti-coagulants and anti- sig. bleeding expected.
- plat activation Bleeding Active PUD platelets: W+
- platelet adhesion, aggregation Bronchospasm Bleeding disorders Max dose: 4g/day
Gout s fx of
Relatively platelet-specific @ low dose Renal failure R (if GFR < 10ml/min) - sulphonylureas
- 75-150mg Gout P, B - methotrexate
Ototoxic in OD: tinnitus
Caution:
- Asthma
- Uncontrolled HTN
Clopidogrel Thienopyridine Bleeding Severe L Avoid
c warfarin. Prodrug converted by
(Plavix) - esp. GI or intracranial B hepatic CYP enzymes.
Irreversible adenosine R antagonist GI upset
- inhibits ADP-induced fibrinogen binding Dyspepsia / PU Used following bare-
to GPIIb/IIIa TTP (rare) metal or drug eluting
Blood dyscrasias (rare) stents.

Stop 7d before surgery if


sig. bleeding expected
Abciximab MAb or synthetic inhibitors of GPIIb/IIIa Bleeding Many, mainly concerned
c Only abciximab can be
Tirofiban Thrombocytopenia bleeding. given PO
Eptifibatide
Given to high-risk pts.
c
NSTEMI
Dipyridamole Phosphodiesterase inhibitor Headache Myasthenia gravis Enhances effects of May be used c aspirin in
- cAMP inhibits plat aggregation adenosine 2O prevention of stroke
TxA2 synthetase inhibitor

Indications for Anti-Platelet Therapy Recommendations Aspirin OD


ACS Clopidogrel is preferred over aspirin + DP-MR for 2O Mixed met acidosis and resp alkalosis
Secondary prevention prevention after stroke Pres: vomiting, dehydration, tinnitus, hyperventilation
IHD Aspirin + DP-MR is preferred over clopi following a TIA. Ix
Stroke Clopidogrel + aspirin is given in the emergent Mx of Salicylate and paracetamol level
TIA STEMI and NSTEMI U+E, ABG, glucose, LFTs, INR.
PVD Continue clopi for 12mo in NSTEMI or 1mo in Rx
Primary prevention STEMI Gastric lavage if <1h since OD
-
10yr CVD risk >20% + controlled BP Clopi + aspirin not beneficial following stroke Correct dehydration and acidosis (c HCO3 ).
Tissue heart valve replacements Alkalinize urine: NaHCO3 + KCl
Haemodialysis: levels >700mg/L, cardiac/renal
failure, seizures

Alasdair Scott, 2012 27


Lipid Lowering Therapy

Drug MOA Side Effects Contraindications Interactions Other


Rosuvastatin Statins Myositis L risk of myositis
c: Monitor
Atorvastatin HMG-CoA Reductase Inhibitors block - stop if CK 5x ULN P (contraception needed - Fibrates - LFTs CK
Simvastatin rate-limiting step in cholesterol synthesis - can rhabdo + ATN during use and for 1mo after) - Macrolides
Lovastatin - Azoles Take statins nocte
Pravastatin hepatocyte cholesterol hepatic Deranged LFTs - Grapefruit juice (ALS) - cholesterol synth
LDL receptors LDL cholesterol GI upset - Protease Inhibitors overnight
(ing potency) - Ciclosporin
HDL - Nicotinic Acid
TGs (mild)
Mild W+
Bezafibrate Fibrates Gallstones Gallbladder disease risk of myositis
c statins Use
Gemfibrozil Stimulate lipoprotein lipase GI upset PBC s fx of anti-diabetics - hypertriglyceridaemia
- TG: 36% appetite albumin
- LDL: 8% Myositis - (esp. nephrotic syndrome) W+
- HDL: 10% Blood dyscrasias RLPB
Cholestyramine Anion Exchange Resin GI upset Complete biliary obstruction s absorption of other Dont take w/i ~3h of
Cholestipol Binds bile acids and prevents - bloating drugs: e.g. digoxin other drugs.
enterohepatic recycling - constipation
liver must synthesise more bile acids - N/V
from cholesterol LDL receptors Can TGs
Impair absorption of fat
sol drugs / vits (ADEK)
Nicotinic Acid Inhibits cholesterol and TG synthesis Flushing Peptic ulcer
HDL n/v
Ezetimibe Inhibits intestinal cholesterol absorption GI upset risk of myositis
c statins
Pro-drug Myalgia
Omega-3 FAs TGs GI upset
Orlistat Pancreatic lipase inhibitor GI upset Cholestasis Warfarin Use
impaired absorption of dietary fat - steatorrhoea - Difficulty controlling INR - adjunct in obesity Mx
- abdo distension
absorption of fat-sol
drugs and vitamins

Statin Indications
Any known CVD
DM (age >40)
10yr CVD risk 20%
Aim: TC 4mM

Alasdair Scott, 2012 28


Warfarin
MOA INR Target and Duration
Inhibits Vit K epoxide reductase
Prevents recycling of Vit K functional Vit K deficiency Indication INR Duration
Inhibits synthesis of factors 2, 7, 9, 10, C and S DVT prophylaxis 2-2.5
Initially procoagulant: protein S is depleted first. Calf DVT 2.5 Cause known: 6wks
No cause: 3mo
Indications Above knee DVT 2.5 Cause known: 3mo
No cause: 6mo
Treatment Prophylaxis PE 2.5 Cause known: 3mo
No cause: 6mo
VTE VTE
Recurrent DVT/PE 2.5 Indefinite
AF
3.5 if on W
Mechanical heart valves
AF 2.5 Indefinite
Large anterior MI (for 3mo)
Mitral valve 2.5 Indefinite
Dilated cardiomyopathy / LV aneurysm disease
NB. Pts.
c Ca-assoc. VTE should initially be treated for 6mo Anti-phospholipid 3.5 Indefinite
with therapeutic dose of LMWH rather than warfarin. syn.
Metal valves 3.5 Indefinite
Pharmacokinetics
Dosing: Tait Model
Long t: 40hrs
Day 1-4: warfarin 5mg OD @ 6pm
Takes 16hrs to affect INR
Day 5: check INR and adjust dose according to table
Peak INR effect of a dose seen @ 2-3d
Day 8: check INR and adjust dose according to table
Effect of a given dose lasts 4-5d
>Day 8: check INR every 4 days and dose accordingly
Highly albumin-bound
OPD: warfarinisation over 3-4wks is safe and effective
CyP metabolism

Side Effects Raised INR


Haemorrhage, bruising
Major bleeding
Skin necrosis (due to protein S deficiency)
Stop warfarin
Purple toe syndrome (cholesterol embolism)
Vit K IV
Osteoporosis
Prothrombin complex concentrate
Hepatic dysfunction FFP 15ml/kg if unavailable

Caution >8 (no bleed or minor bleeding)


Hepatic impairment: avoid if severe Stop warfarin and restart when INR <5
Renal impairment: avoid if severe Vit K IV/PO if risk factors for bleeding
Alcoholics E.g. head injury, stroke, epistaxis, prev Hx

Contraindications 6 8 (no bleed or minor bleeding)


Pregnancy Stop warfarin and restart when INR <5
Teratogenic in 1st trimester
Foetal haemorrhage in 3rd trimester 4.5 6
PUD or omit warfarin and restart when INR <5
Severe HTN
Caution if R/L, recent surgery, risk of falls Vitamin K
Onset of action = 6h
Interactions Oral is as efficacious as IV
Oral Vit K can prolonged anticoagulant resistance
If continuing anticoagulation, avoid if possible
W+ W-
Continuing warfarin: 0.5mg slow IV
Enzyme inhibitors CBZ
Discontinuing warfarin: 2.5-5mg IV
EtOH Rifampicin
Simvastatin OCP PCC
NSAIDs Phenytoin Factors 2, 7, 9 and 10
Dipyridamole Barbiturates
Immediate reversal of anticoagulation
Amiodarone St. Johns Wart
Temporary effect: give c
Vit K
Abx (may also )
Risk of VTE and v. expensive
Cranberry juice
VTE Prophylaxis on Long-Haul Flights
Low risk: avoid dehydration, regularly flex ankles
Mod risk: as above + compression travel socks
Previous VTE, GA w/i last 1-2mo
High risk: as above + consider LMWH before flight
Surgery under GA w/i last 1mo
Alasdair Scott, 2012 29
Heparin Thrombolytics
MOA MOA Indications
Co-factor for ATIII: inhibits factors 2, 10, 11 and 12. Convert plasminogen plasmin STEMI
LMWH and fondaparinux only inhibit factor 10. Plasmin breaks down fibrin Stroke
Life-threatening PE
LMWH or UH Acute limb ischaemia
LMWH has longer t and response is more predictable no monitoring needed
UH has rapid onset and short t Streptokinase (derived from streptococci)
Useful when rapid control over effects needed (e.g. risk of bleeding)
Less risk of HIT and osteoporosis
c LMWH Side Effects
Bleeding
Indications Allergic reactions
Rash
Anaphylaxis
Treatment Prophylaxis
Reperfusion dysrhythmias after MI
VTE VTE
Hypotension
ACS Coagulation in extra-corporeal circuits
Acute arterial obstruction Development of Abs: only use streptokinase once

Side effects Admin: infusion over 1h


Thrombocytopenia
Immune-mediated
Develops ~6d after initiation (2-3% of people
c UH) Rh-TPA
thrombosis
Osteoporosis (long-term use) Side Effects
Hyperkalaemia: heparin inhibits aldosterone Bleeding, hypotension, reperfusion dysrhythmias

Contraindications Admin
Tenectaplase, reteplase: bolus
Bleeding disorders Cerebral haemorrhage
Alteplase: infusion
Plats <60 Severe HTN
Give
c UH heparin IV for 24-48 to avoid rebound hypercoaguable state.
Previous HIT Neurosurgery
PU

Contraindications
Dosing
LMWH: e.g. enoxaparin Absolute Relative
Prophylaxis: 20-40mg pre- and post-surgery Haemorrhagic stroke at any time TIA in last 6mo
Treatment: 1.5mg/kg/24h Ischaemic stroke in last 6mo Warfarin
UH CNS trauma or neoplasms Pregnancy or w/i 1wk post-partum
5000iu bolus IV over 30min Major trauma/surgery in last 3wks Refractory resuscitation
Infuse UF @ 18iu/kg/h GI bleed w/i last 1mo Refractory HTN (>180/110)
Check APTT @ 6h (aim for 1.5-2.5x control) Known bleeding disorders Advanced liver disease
Aortic dissection Infective endocarditis
Non-compressible puncture (e.g. LP) Acute peptic ulcer

Alasdair Scott, 2012 30


Atrial Fibrillation Angina Pectoris
Acute (<48h) Lifestyle
Unstable emergency cardioversion Stop smoking
Control ventricular rate: -B or CCB Wt. loss and exercise
Start LMWH Healthy diet: oily fish, fruit, veg, sat fats, Na
Cardiovert: electrical or flecainide / amiodarone

Paroxysmal AF Medical
Self-limiting, <7d, recurs
Anticoagulate: use CHADSVAS 2O Prevention: prevent cardiovascular events
Rx pill-in-pocket : flecainide, propafenone Aspirin 75mg OD
Prevention: -B, sotalol or amiodarone ACEi (esp. if angina + DM)
Statins: simvastatin 40mg
Persistent AF Antihypertensives
>7d, may recur even after cardioversion
Anti-anginals: prevents angina episodes
Try rhythm control first-line if 1. GTN (spray or SL) + either
st
Symptomatic or CCF 1 : -B (e.g. Atenolol 50-100mg OD)
Younger (<65) 2nd: CCB (e.g. Verapamil 80mg TDS)
Presenting first time
c lone AF 2. If either -B or CCB doesnt control symptoms, try the
other option.
Secondary to treated precipitant
3. Can try -B + dihydropyridine CCB
e.g. amlodipine 10mg/24h
Rhythm Control
4. If symptoms still not controlled
TTE first: structural abnormalities
ISMN 20-40mg BD (8h washout @ PM) or slow-
Anticoagulate c warfarin for 3wks
release nitrate (Imdur 60mg OD)
or use TOE to exclude intracardiac thrombus.
Ivabradine (esp. if cant take -B)
Pre-Rx 4wks c sotalol or amiodarone if risk of failure
Nicorandil 10-30mg BD
Electrical or pharmacological cardioversion Ranolazine
4 wks anticoagulation afterwards (target INR 2.5)

Maintenance antiarrhythmic
Interventional: PCI
Not needed if successfully treated precipitant
st
1 : -B (e.g. bisoprolol, metoprolol). Indications
2nd: amiodarone Poor response to medical Rx
Refractory angina but not suitable for CABG
Rate control (target <90bpm at rest):
1st line: -B or rate-limiting CCB (NOT both!) Complications
2nd line: add digoxin (dont use as monotherapy) Restenosis (20-30% @6mo)
3rd line: consider amiodarone Emergency CABG (<2%)
MI (<2%)
Mx of Permanent AF Death (<0.5%)
Failed cardioversion / unlikely to succeed
AF >1yr, valve disease, poor LV function Clopidogrel s risk of restenosis
Pt. doesnt want cardioversion Bare metal stent: 1mo
Rate control Drug-eluting (e.g. sirolimus) stent: 1yr

CHA2-DS2-VAS Score
Determines necessity of anticoagulation in AF Surgical
Warfarin CI in AF CABG
Bleeding diathesis, plats, BP > 160/90, poor
compliance
Dabigatran may be cost-effective alternative.

CHA2-DS2 VAS
CCF Vascular disease
HTN Age: 65-74yrs
Age 75 (2 points) Sex: female
DM
Stroke or TIA (2 points)

Score
0: aspirin 300mg
1: Warfarin

Alasdair Scott, 2012 31


Chronic Heart Failure Hypertension
General Mx Do ABPM to confirm Dx before Rx (unless severe HTN)

1O/2O Cardiovascular Risk Lifestyle interventions


Stop smoking exercise
salt intake smoking, EtOH, salt, caffeine
Optimise wt.: or - dietician
Supervised group exercised based rehab programme Indications for Pharmacological Rx
Aspirin <80yrs, stage 1 HTN (>140/90) and one of:
Statin Target organ damage (e.g. LVH, retinopathy)
10yr CV risk 20%
Rx Precipitants / Causes Established CVD
Underlying cause DM
Valve disease Renal disease
Arrhythmias Anyone with stage 2 HTN (>160/100)
Ischaemia Severe / malignant HTN (specialist referral)
Exacerbating factors Consider specialist opinion if <40yrs with stage 1 HTN
Anaemia and no end organ damage.
Infection
BP BP Targets
Under 80yrs: <140/90 (<130/80 in DM)
Over 80yrs: <150/90
Specific Mx
ACEi, -B and spiro mortality CV Risk Mx
O
Statins for 1 prevention if 10yr CVD risk 20%
1st line: ACEi/ARB + -B + loop diuretic Aspirin may be indicated: evaluate risk of bleeding
ACEi/ARB: e.g. lisinorpil or candesartan
Hydralazine + ISDN if not tolerated
Antihypertensive Rx
-B: e.g. carvedilol or bisoprolol (licensed)
Start low, go slow < 55 > 55 / Black
E.g. carvedilol 3.125mg/12h 25-50mg/24h
Wait 2wks between increments 1: A C (or D)
Switch stable pts taking a -B for a comorbidity to
a -B licensed for heart failure
-B therapy may be particularly good in COPD 2: A + C (/D)

Loop diuretic: frusemide or bumetanide


nd 3: A+C+D
2 line: get specialist advice
Spironolactone / eplerenone
Watch K carefully (on ACEi too)
4: Resistant HTN
ACEi + ARB A+C+D+ consider further diuretic (e.g.
spiro) or -blocker or -B.
Vasodilators: hydralazine + ISDN Seek expert opinion
Additional Rx in Blacks
rd
3 line A: ACEi or ARB
Digoxin e.g. lisinopril 10mg OD ( to 30-40mg)
Cardiac resynchronisation therapy ICD e.g. candesartan 8mg OD (max 32mg OD)
C: CCB: e.g. nifedipine MR 30-60mg OD
D: Thiazide-like diuretic: e.g. chlortalidone 25-50mg OD
Other Considerations
Monitoring In step 2, use ARB over ACEi in blacks.
BP: may be v. low Avoid thiazides + -B if possible: risk of DM
Renal function Only consider -B if young and ACEi/ARB not tolerated.
Plasma K
Daily wt.
Use amlodipine for comorbid HTN or angina Malignant HTN
Avoid verapamil, diltiazem and nifedipine (short BP > 180/110 + papilloedema and/or retinal
acting) haemorrhage
Controlled in BP over days to avoid stroke
Atenolol or long-acting CCB PO
Encephalopathy / CCF: fruse + labetalol / nitroprusside IV
Aim for 110 diastolic over ~4h
Alasdair Scott, 2012 32
Respiratory
Contents
Bronchodilators............................................................................................................................................................................ 34
Inhaled Corticosteroids ................................................................................................................................................................ 34
Miscellaneous Respiratory Drugs................................................................................................................................................ 35
Chronic Asthma ........................................................................................................................................................................... 36
Chronic COPD ............................................................................................................................................................................. 36

Alasdair Scott, 2012 33


Bronchodilators

Drug MOA Side Effects Contraindications Interactions Other


-agonists Act @ bronchial 2 receptors Tachycardia K in high doses c Salbutamol may be
- SM relaxation Tremor - Corticosteroids given IVI in acute severe
Short-Acting, Fast Onset - mucus secretion - Loop / thiazide diuretics asthma.
- 2-4hrs - Theophylline
- Salbutamol: Ventolin
- Terbutaline: Bricanyl

Long-Acting
- 12-18hrs
- Salmeterol: Serevent
- Formoterol (Fast onset)
Muscarinic Antagonists Bronchodilatation Dry mouth Caution
mucus secretion - Closed Angle Glaucoma
Short-Acting - Prostatic hypertrophy
- 3-6hrs
- Ipratropium: Atrovent

Long-Acting
- Tiotropium: Spiriva

Inhaled Corticosteroids

Drug MOA Side Effects Contraindications Interactions Other


Beclometasone: Becotide Act over wks inflammation Oral candidiasis risk of complications
Budesonide: Pulmicort - cytokine production - use a spacer
Fluticasone: Flixotide - prostaglandin / leukotriene High doses may typical - rinse mouth after use
synthesis steroid SEs
Symbicort - IgE secretion Fluticasone is ~2x as potent
- Budesonide + formoterol - leukocyte recruitment as beclo or budes
- use lower dose
Seretide Prevent long-term in lung function
- Fluticasone + salmeterol Symbicort can be used as a
preventer or reliever
because of formoterols fast
onset.

Alasdair Scott, 2012 34


Miscellaneous Respiratory Drugs

Drug MOA Side Effects Contraindications Interactions Other


Theophylline MR Methylxanthines Nausea d levels Aminophylline is IV form
Aminophylline Arrhythmias - smoking - give IVI slowly
PDE inhibitors Seizures - EtOH - Too fast VT
- cAMP bronchodilatation K - CyP inducers - monitor c ECG and
check plasma levels
d levels
- CCBs CyP metabolism
- CyP inhibitors
Montelukast: Singulair Leukotriene receptor antagonist ? Churg-Strauss Particularly useful for NSAID-
Zafirlukast - Block cysteinyl leukotrienes and exercise-induced asthma

Roflumilast PDE-4 inhibitor GI Sev immunological


disease
Omalizumab Humanised anti-IgE mAb SC injection every 2-4wks
Used for severe asthma
Carbocysteine Mucolytic GI bleed (rare) Active peptic ulceration Use: COPD
Dornase ALFA (DNase) Mucolytic Use: CF
Non-sedating Selective H1 R inverse agonists Hypotension CI: sev hepatic disease
- Certirizine - aka H1 antagonists Arrhythmia: QT
- Des-/ Loratidine: Clarityn Caution
- Fexofenadine Older agents - Long QT
- Drowsiness - BPH
Sedating - Anti-AChM - Closed-angle glaucoma
- Chlorphenamine: Piriton

Alasdair Scott, 2012 35


Chronic Asthma Chronic COPD
General Measures: TAME Assess Severity
Technique for inhaler use Mild: FEV1 >80% (but FEV/FVC <0.7 and symptomatic)
Avoidance: allergens, smoke (ing), dust Mod: FEV1 50-79%
Monitor: Peak flow diary (2x/d) Severe: FEV1 30-49%
Written instruction based on peak flow Very Severe: FEV1 < 30%
Educate
Liaise c specialist nurse General Measures
Need for Rx compliance Stop smoking
Emergency action plan Specialist nurse
Nicotine replacement therapy
Drug Ladder Bupropion, varenicline (partial nicotinic agonist)
Support programme
1 SABA PRN Pulmonary rehabilitation / exercise
If use >1/d or nocte symptoms step 2 Rx poor nutrition and obesity
Screen and Mx comorbidities
2 Low-dose inhaled steroid: beclometasone 100-400ug bd e.g. cardiovasc, lung Ca, osteoporosis
200ug bd is good starting dose for most Depression
Influenza and pneumococcal vaccine
3 LABA: salmeterol 50ug bd Review 1-2x/yr
Good response: continue Air travel risky if FEV1<50%
Benefit but control still poor: steroid to 400ug bd
No benefit: discontinue + steroid to 400ug bd Mucolytics
Consider if chronic productive cough
If control is still poor consider trial of: E.g. Carbocisteine (CI in PUD)
Leukotriene receptor antagonist (e.g. monetelukast)
Esp. if exercise- or NSAID-induced asthma Breathlessness and/or exercise limitation
MR Theophylline SABA and/or SAMA (ipratropium) PRN
SABA PRN may continue at all stages
4 Trials of
inhaled steroid to up to 1000ug bd Exacerbations or persistent breathlessness
Leukotriene receptor antagonist FEV1 50%: LABA or LAMA (tiotropium) (stop SAMA)
MR Theophylline FEV1 <50%: LABA+ICS combo or LAMA
MR agonist PO
Persistent exacerbations or breathlessness
5 Oral steroids: e.g. prednisolone 5-10mg od LABA+LAMA+ICS
Use lowest dose necessary for symptom control 500ug fluticasone BD
Maintain high-dose inhaled steroid 1000ug beclometasone BD
Refer to asthma clinic Roflumilast / theophylline (PDIs) may be considered
Consider home nebs

LTOT
Aim: PaO2 8 for 15h / day ( survival by 50%)
Clinically stable non-smokers
c PaO2 <7.3 (stable on two
occasions >3wks apart)
PaO2 7.3 8 + PHT / cor pulmonale / polycythaemia /
nocturnal hypoxaemia
Terminally ill pts.

Surgery
Recurrent pneumothoraces
Isolated bullous disease
Lung volume reduction

Alasdair Scott, 2012 36


Central Nervous System
Contents
Anti-Parkinsonian Drugs .............................................................................................................................................................. 38
Anti-Epileptic Drugs ..................................................................................................................................................................... 39
Anti-Migraine ............................................................................................................................................................................... 40
Multiple Sclerosis Therapy .......................................................................................................................................................... 41
Anti-Emetics ................................................................................................................................................................................ 42
Anti-Depressants ......................................................................................................................................................................... 43
Analgesia ..................................................................................................................................................................................... 44
Other Psychiatric Drugs .............................................................................................................................................................. 45
Migraine ....................................................................................................................................................................................... 46
Epilepsy ....................................................................................................................................................................................... 46
Parkinsons Disease .................................................................................................................................................................... 47
Drug-induced Movement Disorders............................................................................................................................................. 47
Pain Control ................................................................................................................................................................................. 48
Multiple Sclerosis......................................................................................................................................................................... 49
Myasthenia Gravis ....................................................................................................................................................................... 49
Psychiatry .................................................................................................................................................................................... 50

Alasdair Scott, 2012 37


Anti-Parkinsonian Drugs

Drug MOA Side Effects Contraindications Interactions Other


Levodopa Dopamine Pro-drug Dyskinesia Glaucoma (closed) fx d by antipsychotics Always give
c peripheral dopa-
On-Off phenomena MAOIs decarboxylase inhibitor
Crosses BBB and converted to Da Psychosis Melanoma HTN crisis
c non-selective - Carbidopa (Co-careldopa)
by dopa-decarboxylase ABP MAOIs - Benserezide (Co-beneldopa)
Mouth dryness Loss of response w/i 2-5yrs
Insomnia Anti-HTNs enhance BP fx
N/V Give domperidone for n/v
EDS Food (protein) affects
absorption Short t at least TDS dosing
Apomorphine Non-selective Da agonist V. emetogenic Only give SC
- give domperidone for 2
days before starting Rx Rescue pen for off freezing
Injection site reactions
Bromocriptine Ergot-derived Da agonists Fibrosis Cardiovascular disease Levels d by: Not often used in parkinsonism
Cabergoline Vasospasm: cardiac, digital Porphyria - octreotide due to SEs
Pergolide GI upset Psychosis - macrolides
Postural hypotension
Drowsiness
Neuropsych syndromes
Ropinirole Synthetic Da agonists GI upset
Rotigotine Drowsiness
Pramipexole Postural hypotension
Neuropsych syndromes
Selegiline Selective MAO-B inhibitors GI upset Used alone to delay need for L-
Rasagiline Prevent intraneuronal degradation Insomnia (selegiline) dopa
of Da Postural hypotension Adjunct to L-dopa to end-of-
Buccal preps better bioavailability (no cheese reaction) dose effects
Entacapone COMT inhibitors Reddish-brown urine Interact
c off period of L-DOPA
Tolcapone GI disturbance sympathomimetics
Inhibit peripheral Da degradation Dyskinesias Tolcapone has better efficacy
Tolcapone hepatotoxic but requires LFT monitoring.
Amantadine Da release GI upset Gastric ulcer May be used in PD for late-
Weak anti-cholinergic Sleep disturbance Epilepsy onset dyskinesia
Livedo reticularis
Neuropsych syndromes
Procyclidine Muscarinic antagonists Anti-AChM Useful in drug-induced
Benzhexol Memory impairment parkinsonism and mild PD in
Reduce tremor Confusion young pts: esp. tremor

L-DOPA Motor Fluctuations


Peak dose dyskinesias
End-of-dose dyskinesia / akinesia: deterioration as dose wears off c progressively shorter benefit.
On-Off effect: unpredictable fluctuations in motor performance unrelated to timing of dose.
Alasdair Scott, 2012 38
Anti-Epileptic Drugs

Drug MOA Side Effects Contraindications Interactions Other


Valproate Na+ channel blockers GI upset Acute porphyria fx d by:
st O
1 line for 1 generalised
Use-dependent Hepatotoxicity - antimalarials seizures
Inhibit action potential generation Personal/fam hx of - antidepressants
Appetite wt. severe liver dysfunction - antipsychotics CYP inhibitor
Liver failure - some anti-epileptics
Pancreatitis L/P Monitor
Reversible hair loss levels d by cimetidine - FBC, LFTs
Oedema
Ataxia s fx of: Most teratogenic AED
Teratogenicity, Tremor, - aspirin - 6% overall
Thrombocytopenia - lamotrigine
Encephalopathy: due to - warfarin
ammonia

Carbamazepine Skin reactions (e.g. SJS) Unpaced AV conduction s fx of: Has active metabolite produced
- Tegretol Blood dyscrasias (WCC) defects - COCP in the liver
Na+ (SIADH) - Doxy
Foetal NTDs Hx of BM depression - corticosteroids CYP inducer
GI upset - anti-epileptics (inc. CBZ)
Porphyria - nifedipine Monitor
Dose-related - Warfarin - se levels
- Dizziness/vertigo MAOIs - U+Es, LFTs, FBC
- Ataxia levels d by:
- Diplopia - macrolides
Phenytoin Acute Dont give IV if cardiac - cimetidine V. albumin bound
- Drowsiness dysrhythmias - diltiazem and verapamil
- Cerebellar fx (DANISH) - EtOH CYP inducer
- Rash Caution: DM, BP, L/H, - NSAIDs
P (cleft palate) - esomeprazole Saturable kinetics
Chronic - dose t
- Gingival hyperplasia levels d by:
- Hirsutism + acne - rifampicin Monitor
- folate - antipsychotics - FBC
Lamotrigine Inhibits glutamate release Rashes (SJS, TEN, lupus) L fx d by: Monitor
Cerebellar fx - OCP - U+Es, LFTs, FBS, clotting
Blood dyscrasias - phenytoin, CBZ
Hepatotoxic - TCAs and SSRIs Stop if any sign of a rash

levels d by valproate Safest drug for pregnancy


Ethosuximide Ca2+ channel blocker GI upset ?makes tonic-clonics Only used for childhood
worse absence seizures
Vigabatrin Visual field defects

Alasdair Scott, 2012 39


Anti-Migraine

Drug MOA Side Effects Contraindications Interactions Other


Rizatriptan 5HT1B/1D receptor agonist Sensations of tingling/heat/ IHD risk of CNS toxicity
c Used for Rx of acute attacks
Sumatriptan tightness/pressure Coronary vasospasm SSRIs
Reverses dilatation of cerebral PVD Dont use >2-3x / wk
vessels Dizziness Hx of MI, CVA, TIA - chronic migraine
Flushing HTN (mod/sev)
Ergotamine Partial 5HT1 receptor agonist GI upset IHD Use limited by SEs
Dizziness Coronary vasospasm
PVD
HTN (mod/sev)
Hx of MI, CVA, TIA
Pizotifen 5HT2 receptor antagonist and Drowsiness Prophylaxis
antihistamine appetite and wt.
Amitriptylline 5HT and NA reuptake inhibitor Anti-cholinergic Recent MI (w/i 3mo) MAOIs HTN and CNS Hepatic metabolism
Anti-adrenergic Heart block excitation
Anti-histamine L Prophylaxis
Levels d by
OD: prolonged QT TDP - SSRIs
- Cimetidine

risk of arrhythmias
c
amiodarone.

Alasdair Scott, 2012 40


Multiple Sclerosis Therapy
Drug MOA Side Effects Contraindications Interactions Other
Methylpred Inhibits PLA2 PG and PAF Cushings High dose (up to 1g/day) for
PMN extravasation PMN in blood - DM acute flares.
Lymphopaenia - Central obesity
Phagocytosis - Dyslipidaemia Short course (3-5d ays)
Ab production - PUD
cytokine and proteolytic enzyme - Osteoporosis
- Hirsutism, acne
Interferon-1 Flu-like symptoms Decompensated L Relapsing remitting or
Injection site reaction Severe depression secondary progressive MS

Monitor for hepatotoxicity


Glatiramer Random polymer of amino acids found Flu-like symptoms Relapsing remitting MS
in myelin basic protein Injection site reaction
? acts as decoy
Natalizumab anti-4 integrin
Alemtuzumab anti-CD52
Baclofen GABAB agonist Sedation PUD fx d by TCAs Rx painful muscle spasms
tone
Skeletal muscle relaxant Nausea Dont withdraw abruptly
Urinary disturbance - hyperthermia
- spasticity
Dantrolene Prevents Ca2+ release from Hepatotoxicity L
sarcoplasmic reticulum GI upset

Skeletal muscle relaxant


Oxybutynin Antimuscarinic Dry mouth Myasthenia Used for detrusor instability
GI upset GI/bladder obstruction
Blurred vision

Alasdair Scott, 2012 41


Anti-Emetics
Drug MOA Side Effects Contraindications Interactions Other
Metoclopramide D2-receptor antagonist EPSEs <20yrs risk of EPSEs c Indications:
Prochlorperazine Prokinetic action in GIT - Dystonias GI obstruction antipsychotics, TCAs and - GI causes esp. GORD
Domperidone - absorption of other drugs - Oculogyric crisis L SSRIs. - Chemo, morning-after-pill, opiates
- PD
Drowsiness Prolactinoma - Migraine
Rash, allergy - Vestibular (prochlorperazine)
prolactin
Domperidone doesnt cross BBB
less EPSEs cf. others
Ondansetron 5HT3-receptor antagonist Constipation Avoid if QTc Levels d by: Indications:
Granisetron Headache - Rifampicin - Post-op
- CBZ, phenytoin - Chemo

Avoid
c drugs that QTc CYP metabolism
Cyclizine H1-receptor antagonist Anti-AChM Severe HF MOAIs can Indications:
Cinnarizine MOAIs antimuscarinic fx - Opioids (but not ACS)
- Vestibular
Hyoscine Anti-muscarinic Anti-muscarinic Glaucoma (closed- s fx of SL GTN Indications:
hydrobromide angle) - Prophylaxis vs. motion sickness
BPH - Hypersalivation
Dexamethasone Steroid unknown anti-emetic effect Indications:
- Chemo (adjunct)
- Surgery
Aprepitant Neurokinin receptor blocker Indications:
- Chemo (adjunct)

Causes of N/V Physiology


Drugs Vomiting regulated by Vomiting Centre and CTZ, both located in the medulla.
Abx: e.g. erythromycin
Anti-parkinsonian: e.g. L-DOPA CTZ
Cytotoxic agents Outside BBB accessible to drugs Emetogenic Receptors:
Opioids Also receives input from vestibular system re. motion H1
Digoxin Expresses: D2 and 5-HT3 receptors D2
GI: GORD, gastro, pancreatitis, obstruction CTZ projects to Vomiting Centre 5HT3
Neoplasia: oesophageal, gastric, duodenal mACh
Vestibular: e.g. labyrinthitis, sea-sickness Vomiting Centre
Neuro: e.g. migraine, ICP Controls visceral and somatic functions involved in vomiting.
Vagal: e.g. pancreatitis, MI, torsion Receives input from CTz
Pregnancy Also receives muscarinic and histaminergic input (H1)

Alasdair Scott, 2012 42


Anti-Depressants

Drug MOA Side Effects Contraindications Interactions Other


Paroxetine SSRI N / V / diarrhoea Active mania P450 inhibitor s levels of Takes 4-6wks for full clinical effect
Citalopram Insomnia Children <18yrs - TCAs Dont stop suddenly
Fluoxetine Headache - except fluoxetine - benzos Avoid w/i 2wks of MAOI
Sertraline Sexual dysfunction - clozapine, haldol
SIADH - CBZ and phenytoin Use
Withdrawal effects - Depression
SSRI + MAOI serotonin syndrome - OCD
- Eating disorders
risk of bleeding
c aspirin - Anxiety disorders
Venlafaxine SNRI HTN HF (3/4) SSRI + MAOI serotonin syndrome 2nd line anti-depressant
GI upset Uncontrolled HTN
QTc risk of bleeding
c aspirin Stop drug if signs of rash
SIADH
Rash
Amitriptyline TCA 1 Recent MI MAOIs HTN and CNS excitation Avoid w/i 2wks of MAOI
Lofepramine - postural hypotension Arrhythmias
Clomipramine Inhibit 5HT and NA uptake - sedation Severe L Levels d by SSRIs
Imipramine Mania
Doxepin H1 risk of arrhythmias
c amiodarone
Norttriptyline - drowsiness Caution
- wt. gain - Glaucoma TCAs lower the seizure threshold
- BPH - effects of AEDs
Anti-AChM
fx of antipsychotics
Arrhythmias
- esp. heart block
Phenelzine MAOI Sedation Phaeo Hypertensive Crisis Moclobemide is reversible and is
Isocarboxacid Hypotension - tyramine containing foods selective for MAOI-A less chance
Moclobemide (A) Inhibit monoamine Anti-AChM - opioids: esp. pethidine of interactions.
Selegiline (B) metabolism
- A: 5HT SSRIs and TCAs serotonin syn.
- B: Da

Serotonin Syndrome TCA Toxicity


Cognitive: headache, agitation, confusion, coma Metabolic acidosis
Autonomic: sweating, HR, palpitations, HTN, hyperthermia Anti-AChM: dilated pupils
Somatic: myoclonus, clonus, hypertonia, tremor CNS: hypertonia, hyperreflexia, extensor plantars, seizures
Cardiac: HR, QTc TDP
Pulmonary: hypoventilation
Monoamine Oxidase Rx: NaHCO3
Metabolises monoamines
MAO-A: adrenaline, norad, serotonin, tyramine, dopamine
MAO-B: dopamine
Alasdair Scott, 2012 43
Analgesia

Drug MOA Side Effects Contraindications Interactions Other


Paracetamol Antipyretic Hepatic failure in OD Severe L
Analgesic
Strong opioids Analgesic effect mediated CNS Avoid in pts.
c acute Rx overdose
c naloxone
- Morphine by u receptor. - Resp. depression respiratory depression
- Diamorphine - Sedation dose in
- Buprenorphine - Nausea / vomiting Head injury - renal impairment
- Fentanyl - Euphoria - cant assess pupils - hepatic impairment
- Pethidine - Meiosis - elderly
- Oxycodone - Anti-tussive
- Dependence
Weak opioid
- Codeine Non-CNS
- Dihydrocodeine - Constipation
- Tramadol - Urinary retention
- Pruritis
- Bradycardia, hypotension
Gabapentin Unknown Sedation fx d by: Pregabalin is more potent
Cerebellar fx - antidepressants gabapentin analogue
Dizziness - antimalarials
Peripheral oedema
Amitriptyline TCAs
Nortroptyline

Alasdair Scott, 2012 44


Other Psychiatric Drugs

Drug MOA Side Effects Contraindications Interactions Other


Li Mood stabiliser Polyuria and polydipsia Hypothyroidism Toxicity d by Monitor
Nephrotoxic P/R/H - NSAIDs - drug levels
GI upset - Diuretics (esp. thiazides) - U+Es
Fine tremor - ACEi / ARB - TFTs
Hypothyroidism
Use
Toxicity - Acute mania
- Coarse tremor - Prophylaxis of BAD
- Cerebellar signs - Resistant depression
- AKI
- Hyper-reflexia toxicity when Na or dehydrated
- Coma - Li reabsorption in renal PCT
Chlorpomazine Typical antipsychotics Sedation fx d by: Monitor FBC, U+E, LFTs
Haldol Da antagonists Anti-AChM - Li
Sulpiride EPSEs - TCAs
Zuclopenthixol Neuroleptic malignant syn.
QTc, postural hypotension
PRL
Sexual dysfunction
wt.
Clozapine Atypical antipsychotics Clozapine: agranulocytosis, wt, DM Can still EPSEs @ high doses
Olanzapine Da antagonists - apart from clozapine
Quetiapine Olanzapine: wt., DM, sedation
Risperidone Clozapine
Quetiapine: sedation - Rx of refractory schizophrenia
- better @ Rx of negative symp
Risperidone: wt,, PRL
Benzodiazepines Promote GABA Sedation Resp depression levels / fx d by: Rx overdose with flumazenil
binding to GABAA Respiratory depression - antipsychotics
receptors Withdrawal - azoles Hepatic metabolism
- macrolides
IV diazepam is given as an emulsion
to risk of thrombophlebitis
Phenobarbitol Potentiate GABAA Sedation See CBZ, phenytoin. CYP inducer
receptors Respiratory depression Primidone is phenobarbital prodrug

Alasdair Scott, 2012 45


Migraine Epilepsy
Acute episode Seizure Type 1st line 2nd line
1st: Paracetamol + metoclopramide / domperidone Generalised Valproate Levitiracetam
2nd: NSAID (e.g. ketoprofen) + M/D - tonic-clonic Lamotrigine
3rd: Rizatriptan (5-HT1B/1D agonist) - absence
CI: IHD, uncontrolled HTN, SSRIs - myoclonic
4th: ergotamine Focal onset Lamotrigine Levitiracetam
Carbamazepine

Prophylaxis
Avoid triggers In Women / Pregnancy
1st: Propanolol, amitriptyline, topiramate Avoid valproate: take lamotrigine (or CBZ)
2nd: Valproate, pizotifen ( wt.), gabapentin 5mg folic acid daily if child-bearing age
CBZ and Phenytoin are enzyme inducers
effectiveness of the OCP
Oral vit K in last month

Driving Advice
Must not drive w/i 12mo of seizures
Pts. who only have seizures while sleeping for 3yrs
can drive.
Person must comply c Rx

Alasdair Scott, 2012 46


Parkinsons Disease Drug-induced Movement Disorders
General Causes
MDT: neurologist, PD nurse, physio, OT, social Neuroleptics
worker, GP and carers Anti-emetics: metoclopramide, prochloperazine
Assess disability L-DOPA acute dystonias
e.g. UPDRS: Unified Parkinsons Disease Rating Scale
Physiotherapy: postural exercises Mx
Depression screening
Disorder Rx
Acute Dystonia Procyclidine
Medical Parkinsonism Procyclidine
Da agonists
Young onset biologically fit Akathisia Propranolol
1. Da agonists: ropinirole, pramipexole Tardive dyskinesia Switch to atypical anti-
2. MOA-B inhibitors: rasagiline, selegiline psychotic
3. L-DOPA: co-careldopa or co-beneldopa Neuroleptic malignant syn. Dantrolene
Da agonists: e.g apomorphine
Biologically frail comorbidities
1. L-DOPA
2. MOA-B inhibitors Other Movement Disorders
Restless legs: ropinerole
Response Fluctuations Benign essential tremor: propranolol
MOA-B inhibitors Focal dystonias: botulinum toxin, benzos
end-of-dose effects Blepharospasm
COMT inhibitor: tolcapone, entacapone Spasmodic torticolis
Lessen off time @ end-of-dose Writers cramp
Apomorphine: potent Da agonist
SC rescue pen for sudden off freezing
Amantidine: weak Da agonist
Rx of drug-induced dyskinesias

Adjunctive Therapies
Domperidone
Rx of drug-induced nausea
Atypical antipsychotics: e.g. quetiapine, clozapine
Disease-induced psychosis
SSRIs: citalopram, sertraline
Depression

Surgical
Interrupt basal ganglia
Deep brain stimulation
Stem cell Transplant

Alasdair Scott, 2012 47


Pain Control
Principles Neuropathic Pain
Oral where possible
Fixed interval to give continuous relief Common Causes
Stepwise approach DM
EtOH
Ca
WHO Analgesia Ladder Trigeminal neuralgia
Post-herpetic neuralgia
1. Non-opioid adjuvants HIV
Paracetamol
NSAIDs
Ibuprofen: 400mg/8h PO Main Options
st
Diclofenac: 50mg PO / 75mg IM 1 : pregabalin / gabapentin
2nd: TCAs
2. Weak opioid + non-opioid adjuvants 3rd: opioids
Codeine 4th: CBZ, valproate, lamotrigine
Dihydrocodeine
Tramadol
Topical: lidocaine patches, capsaicin
3. Strong opioid + non-opioid adjuvants
Morphine: 5-10mg/2h max NB. HIV-assoc. sensory neuropathy responds better to
Oxycodone gabapentin than amitriptyline.
Fentanyl

NSAIDs
gastric and duodenal ulceration
Na and H2O retention
Worsen heart failure
Interfere c ACEi / ARB
May worsen / precipitate asthma
COX-2 selectives CV risk

Potent Opioids
Establish dose c intermediate release preparations then
give maintenance c modified release
Start c oramorph 4-10mg/4h PO c=
breakthrough dose PRN
Switch to modified release (MST) BD: BD dose =
total 24h dose / 2.
Give 1/6 total daily dose as oramorph for
breakthrough pain
Consider PCA

Mx of SEs
Constipation: codanthrusate (stimulant laxative)
Nausea: metoclopramide
Drowsiness: tolerance develops

Other Options
Nerve blocks: visceral pain
Direct local anaesthetic injections: facet joint pain

Adjuvants
Anything not specifically mentioned on the ladder
Surgery
Chemo

Alasdair Scott, 2012 48


Multiple Sclerosis Myasthenia Gravis
MDT: neurologist, radiologist, physio, OT, specialist nurses,
GP, family Diagnosis: Tensilon Test
Edrophonium bolus IV
Positive = improvement of power w/i 1min
Acute Attack
Methylpred 1g IV/PO /24h for 3d
Doesnt influence long-term outcome
Mx
duration and severity of attacks
Symptom Control
Anticholinesterase: e.g. pyridostigmine.
Cholinergic SEs
Preventing Relapse
Immunosuppression
DMARDs
Rx relapses c pred
IFN-: relapses by 30% in relapsing remitting MS
Steroids sparing-agents
Glatiramer: similar efficacy to IFN-
Azathioprine, methotrexate, cyclosporin
Can total remission
Biologicals
Natalizumab: anti-VLA-4 Ab
Thymectomy
Relapses by 2/3 in RRMS
Consider if young onset and disease not control by
Alemtuzumab: anti-CD52
nd anticholinesterases
2 line in RRMS
Always remove a thymoma if present
May be malignant
Remission in 25%, benefit in further 50%.
Symptomatic
Fatigue: modafinil Myasthenic Crisis
Depression: SSRI (citalopram) Plasmapheresis
Pain: amitryptylline, gabapentin IVIg
Spasticity: physio, baclofen, dantrolene, botulinum
Urgency / frequency: oxybutynin, tolterodine
ED: sildenafil
Tremor: clonazepam

Alasdair Scott, 2012 49


Psychiatry
Antidepressants
SSRIs: selective serotonin reuptake inhibitors
Citalopram, sertraline, fluoxetine

SNRIs: serotonin noradrenaline reuptake inhibitors


Venlafaxine

NaSSA: Norad and specific serotonergic antidepressant


Mirtazapine

NRI: Norad reuptake inhibitor


Reboxetine

TCAs: tricyclic antidepressants


Amitriptyline
Imipramine
Clomipramine

MAOIs: monoamine oxidase inhibitors


Meclobemide (RIMA)
Phenelzine

Bipolar Affective Disorder


Lithium (TDM essential)
AEDs: valproate, CBZ
Neuroleptics: olanzapine

Antipsychotics
Typical
Haldol
Chlopromazine

Atypical
Clozapine
Olanzapine
Risperidone
Quetiapine

Alasdair Scott, 2012 50


Infection
Contents
Beta Lactams............................................................................................................................................................................... 52
Inhibitors of Protein Synthesis ..................................................................................................................................................... 53
Other Antibiotics .......................................................................................................................................................................... 54
Anti-Malarials ............................................................................................................................................................................... 55
Anti-Virals .................................................................................................................................................................................... 55
Anti-Retrovirals ............................................................................................................................................................................ 56
Anti-Fungals ................................................................................................................................................................................ 57
Bacterial Infections ...................................................................................................................................................................... 58
Viral Infections ............................................................................................................................................................................. 59
Protozoal Infections ..................................................................................................................................................................... 60
Fungal Infections ......................................................................................................................................................................... 60

Alasdair Scott, 2012 51


Beta Lactams

Drug MOA Use Side Effects CIs Interactions


Penicllins Bactericidal Pen V Streps Hypersensitivity Hypersensitivity - May fx of OCP
- Inhibit bacterial transpeptidase enzyme Pen G (IV) N. meningitidis - Rash 10% x-reactivity
Required for cell wall construction Syphilis - EM c cephs
d by probenecid
Broad Spectrum Pneumococcus - anaphylaxis
- Amoxicillin Listeria
- Ampicillin E. coli GI upset
Enterococci
Penicillinase-resistant MSSA Mac pap rash
c EBV
- Fluclox
Anti-pseudomonal Pseudomonas
- Piperaccillin
- Ticarcillin
Co-amoxiclav Severe CAP
UTI
Tazocin Severe HAP
Sepsis
Cephalosporins Bactericidal 1st gen UTI GI upset Hypersensitivity
- Inhibit bacterial transpeptidase enzyme - Cephalexin AAC
Required for cell wall construction - Cefaclor
2nd gen Mod / sev CAP
Generations have ing activity vs. Gm-ve - Cefuroxime GI sepsis
Pre-op
3rd gen Meningitis
- Cefotaxime Epiglottitis
- Ceftriaxone Gonorrhoea
- Ceftazidime SBP
- Cefixime
Carbapenems Bactericidal Imipenam All Gm+ except MRSA GI upset
- Inhibit bacterial transpeptidase enzyme Meropenam Most Gm-ves
Required for cell wall construction Ertapenam Neutropenic sepsis

V. broad spectrum
- Gm-, Gm+ and anaerobes
- Pseudomonas

Imipenem is rapidly inactivated by the


kidney and must be given c cilastatin
which blocks its metabolism.

Alasdair Scott, 2012 52


Inhibitors of Protein Synthesis

Drug MOA Use Side Effects CIs Interactions Other


Chloramphenicol Bacteriostatic Conjunctivitis Irreversible aplastic PLR
- 50s subunit anaemia
Grey baby syndrome
Gentamicin Aminoglycosides Gm- sepsis Nephrotoxic Myasthenia gravis absorption
c Must monitor levels
Amikacin Neutropenic sepsis Ototoxic - milk - peak and trough
Tobramycin Bactericidal Otitis externa Caution in R - antacids
Neomycin Amino-acyl site of 30s - alter dose and time Must be given IV
Streptomycin subunit Anti-pseudomonal
Tetracycline Tetracyclines COPD exacerbation GI upset Children <12yrs Toxicity by
Doxycycline Acne Hypersenstivity L - frusemide
Bacteriostatic Chlamydia Bone deposition R - cephs
30s subunit Rickettsia - vanc
Brucella - ciclosporin
Lyme disease
Linezolid Oxazolidinones MRSA and VRE Blood dyscrasias Caution in R and L Linezolid is a non-
selective MAOI
Bacteristatic No activity vs. Gm- - avoid SSRI, TCAs
23s component of 50s and Tyramine
subunit
Monitor FBC
Erythromycin Macrolides Pen allergy QTc Caution if QTc P450 inhibitor Also have GI
Clarithromycin Atypical pneumonia Dry skin - W+ prokinetic action
Azithromycin Bacteriostatic Chlamydia Cholestatic hepatitis
50s subunit H. pylori digoxin
Synercid Streptogramins VRE Only used when other
MRSA agents failed.
Bacteriostatic
50s subunit

Clindamycin Lincosamides Active vs. Gm+ cocci and AAC Diarrhoea Stop drug if pt.
bacteroides Hepatotoxicity develops diarrhoea
Bacteriostatic
50s subunit Osteomyelitis
MRSA

Alasdair Scott, 2012 53


Other Antibiotics

Drug MOA Use Side Effects CIs Interactions Other


Vancomycin Glycopeptides Aerobic and anaerobic Gm+ Nephrotoxic dose in renal Must monitor levels
Teicoplanin Bactericidal MRSA Ototoxic impairment - pre-dose trough level
Inhibits cell wall synthesis HAN - tinnitus
Unable to penetrate Gm- Infective endocarditis - SNHL
outer cell wall AAC (PO) Hypersensitivity rash
Poor oral absorption Neutropenia
Ciprofloxacin Fluoroquinilones Broad spectrum: esp. Gm- QTc P P450 inhibitor
Levofloxacin Bactericidal GI infections: campy, shig GI upset
Ofloxacin Inhibit DNA synthesis Pseudomonas: esp. in CF Tendonitis rupture Antacids
Moxifloxacin Prostatitis, PID seizure threshold absorption
Anthrax Photosensitivity
Metronidazole Nitroimidazole Anaerobes Metallic taste Avoid EtOH Aldehyde dehydrogenase
Nitrofurantoin Bactericidal GI sepsis GI upset - Disulfiram-like inhibitor
Tinidazole Inhibits DNA synthesis Aspiration pneumonia Metro: gynaecomastia reaction
AAC
H. pylori
PID
Protozoa:Giardia
Rifampicin Rifamycins Mycobacteria Yellow secretions Jaundice P450 inducer Rifaximin has v. poor oral
Rifaximin Bactercidal Legionella Hepatitis - W- absorption and is used in
Rifabutin Inhibit RNA synthesis Prophylaxis vs. meningits - OCP hepatic encephalopathy.
- AEDs
Trimethoprim Bacteriostatic UTI Blood dyscrasias Severe R and L Stop immediately if rash or
Sulfamethoxazole Folate antagonists PCP EM SJS P dyscrasias occurs.
Sulfadiazine Toxoplasmosis EN
Co-trimoxazole Nephro- + hepato-toxicity
Daptomycin Cell membrane toxin MRSA
- Alternative to linezolid and
syndercid
Colistin Cell membrane toxin Active vs. Gm- Myasthenia gravis
Inhaled for CF
Ethambutol Bacteriostatic Anti-TB Optic neuritis Monitor vision
Inhibits mycobacterial cell - colour vision goes first
wall synthesis
Pyrazinamide Bactericidal Anti-TB Hepatitis Caution in gout Monitor LFTs
Gout
Isoniazid Bacteriostatic Anti-TB Peripheral neuropathy P450 inhibitor risk of SEs if slow
Hepatitis acetylator

Give c pyridoxine
Fusidate Bacteriostatic Active vs. staphs Hepatitis Needs 2nd Abx to prevent
Impetigo (topical) resistance
Blepharitis (topical)
Alasdair Scott, 2012
Osteomyelitis (PO) 54
Anti-Malarials
Drug Use Side Effects CIs Interactions Other
Chloroquine Benign malaria Visual change: rarely retinopathy Caution in G6PD deficiency
Prophylaxis Seizures
EM SJS
Primaquine Benign malaria Haemolysis if G6PD deficient Caution in G6PD deficiency
- eliminate liver stage Methaemoglobinaemia
Malarone Falciparum malaria Abdo pain Avoid in renal impairment if
- proguanil + atovaquoone Prophylaxis Gi upset possible
Mefloquine Prophylaxis Nausea, dizziness Hx of epilepsy or psychosis Avoid if low risk of resistance
Neuropsychiatric signs
Riamet Falciparum malaria QTc Hx of arrhythmias
- artemether + lumefantrine Abdo pain QTc
GI upset Caution in R L

Anti-Virals

Drug MOA Use Side Effects CIs Interactions


Aciclovir Guanosine analogue Genital herpes GI upset Caution in renal
Phosphorylated by viral thymidine kinase Herpes meningitis ARF impairment
Di- and tri-phosphorylated by cellular kinase Herpes zoster Encephalopathy
Aciclovir triphosphate inhibits viral DNA pol Varicella zoster
Aciclovir is poor substrate for host DNA pol and TK Bells palsy
Valaciclovir Aciclovir prodrug
Converted aciclovir by hepatic esterases during 1st pass
metabolism
Better oral bioavailability
Famciclovir Pro-drug c same MOA as aciclovir
Ganciclovir 2-deoxyguanosine analogue CMV Rx BM suppression risk of BM suppression
Phosphorylated to dGTD analogue by viral UL97 - retinitis c zidovudine

Triphosphate competitively inhibits viral DNA pol - pneumonitis
IV only CMV prophylaxis
Valganciclovir Ganciclovir prodrug with better oral bioavailabilty HHV-6 Tx disease
Foscarnet Binds to pyrophosphate-binding site and inhibits viral DNA pol CMV Rx Nephrotoxic Renal Tx
Doesnt require viral TK - avoid in renal Tx
IV only
Cidofovir Inhibits viral DNA polymerase Resistant CMV Nephrotoxic
No activation required infections

Alasdair Scott, 2012 55


Anti-Retrovirals

Class Antiviral MOA Side Effects Interactions


NRTI Emtricitabine NucleoSide Reverse Transcriptase Inhibitors Hepatitis
Stavudine - stop if LFTs
Tenofovir Except Tenofovir which is nucleoTide reverse transcriptase inhibitor Lactic Acidosis (type B)
Abacavir Painful peripheral neuropathy
Didanosine Rash
Lamivudine GI disturbance
Zidovudine
PIs Ritonavir Inhibit viral protease required for virus assembly Metabolic syndrome P450 inhibitors
Ritonavir is used to boost levels of other PIs Lipodystrophy
Indinavir

Saquinavir

Lopinavir / ritonavir
Kaletra
NNRTI Efavirenz Non-competitive inhibition of reverse transcriptase Insomnia, vivid dreams,
Nevirapine NB. Nevirapine is used to prevent HIV transmission during pregnancy dizziness
EM SJS
Integrase Raltegravir Inhibit integration of transcribed viral DNA into host genome
inhibitors Elvitegravir
CCR5 inhibitor Maraviroc Binds CCR5 preventing interaction with gp120
Inhibits attachment of HIV
Fusion inhibitor Enfuviritide Binds gp41 and inhibits fusion Hypersensitivity at injection site

Indications Lipodystrophy
CD4 350 Fat redistribution
AIDS-defining illness SC fat
Pregnancy abdo fat
HIVAN Buffalo hump
Co-infected
c HBV when Rx is indicated for HBV Insulin resistance
Dyslipidaemia
Use 2 NRTIs + 1 NNRTI or PI

Immune Reconstitution Inflammatory Syndrome


Improvement in immune function 2O to ARV Rx
Marked inflammatory reaction vs. residual opportunistic organisms
Paradoxical worsening of symptoms on initiation of ARVs

Alasdair Scott, 2012 56


Anti-Fungals

Class Drug Mechanism Indication SEs Misc


Polyenes Amphotericin B Interacts with ergosterol pore formation Severe systemic fungal infections (IV) Nephrotoxic (IV) Monitor Cr
- cryptococcal meningitis
Fungicidal - pulm. Aspergillosis IV reaction (after 1-3h) PO version is
- systemic candidiasis - fever non-toxic
- hypotension
- nausea/vomiting
Nystatin Candidiasis: cutaneous, vaginal, Toxic if given IV PO or topical
mucosal, oesophageal
Imidazoles Ketoconazole Blocks ergosterol synth by inhibiting 14- Chronic mucocutaneaous candidiasis Hepatotoxic
demethylase membrane fluidity androgen synthesis
Miconazole Dermatophyte infections
Clotrimazole Inhibits replication Mucocutaneous candidiasis
Triazoles Fluconazole Oral/vag/oesophagus candida P450 inhibitor
Prevents hyphae formation Alternative to ampho B for systemic
infections
Itraconazole Broad spectrum Blasto/histo/coccidio
Sporotrichosis
Fungistatic Chromomycosis
Aspergillus
Voriconazole Invasive candida or aspergillus in Photophobia
immunocomp Rash
BMT transplant pt. prophylaxis Hepatotoxic
Posaconazole Invasive candida, mucor and
aspergillus in immunocomps
Allylamines Terbinafine Blocks ergosterol synth by inhibiting Dermatophyte Infections GI effects
squalene epoxidase membrane disruption Hives
LFTs
Fungicidal Reversible agranulocytosis
Echinocandins Caspofungin Inhibit -glucan synthesis Invasive aspergillosis or candidiasis V. low toxicity IV only

Fungicidal vs. yeasts Empiric Rx for fungal infection in febrile GI upset


neutropenia Hypersensitivity
Fungistatic vs. moulds
Flucytosine Inhibits DNA/RNA synthesis Crptococcal meningitis (in combo with Bone marrow suppression
amphotericin B) Deranged LFTs
Griseofulvin Disrupts spindle formation in mitosis Dermatophyte infections of Very slow acting
skin/hair/nails

Alasdair Scott, 2012 57


Bacterial Infections
Pneumonias Meningitis
Community: benpen 1.2g IV/IM
CAP <50: ceftriaxone 2g IVI/IM BD
>50: ceftriaxone + ampicillin 2g IVI /4h
Mild amoxicillin 500mg TDS PO for 7d or Viral suspected: aciclovir
clarithro 500mg BD PO for 7d Give Dex 0.15mg/kg/6h c first dose of Abx unless in
septic shock or meningococcal sepsis
Mod amoxicillin 500mg TDS and clarithro 500mg BD PO/IV
for 7d (clarithro alone if pen allergy)
Urinary Tract Infection
Sev Co-amoxiclav 1.2g TDS IV / cefuroxime 1.5g TDS IV Pyelonephritis: cefotaxime 1g IV BD for 10d
and clarithro 500mg BD IV for 7-10d Lower UTI: Rx for 7d
Add fluclox if staph suspected. 1st: Trimethoprim 200mg BD
1st: Nitrofurantoin 50mg QDS (avoid in eGFR)
Atyp Chlamydia: tetracycline 2nd: Cephalexin 500mg BD
PCP: Co-trimoxazole 2nd: Amoxicillin 500mg TDS
Legionella: Clarithro + rifampicin Prostatitis: Cipro 500 mg BD for 28d

HAP
Mild / <5d: Co-amoxiclav 625mg PO TDS for 7d Genital Tract
Severe / >5d: Tazocin for 7d Chlamydia / NSU: azithromycin 1g STAT
vanc for MRSA Gonorrhoea
gent for Pseudomonas Azithromycin 1g STAT + ceftriaxone 500mg IM
PID: ofloxacin + metronidazole
Aspiration Pneumonia
Co-amoxiclav 625mg PO TDS for 7d
GIT
Exacerbation of COPD GI sepsis: cefuroxime + metronidazole
Rx if sputum purulence
c sputum volume or Campylobacter: ciprofloxacin
dyspnoea or consolidation on CXR Shigella: ciprofloxacin
Amoxicillin 500mg PO TDS for 7d C. diff
Or, doxy 200mg STAT + 100mg BD for 7d 1st: metronidazole PO
2nd: Vancomycin PO
Legionella
Levofloxacin or,
Clarithromycin + rifampicin Sepsis
Tazocin
PCP Vanc if MRI suspected
st
1 : co-trimoxazole Gent for gram -ves
2nd: pentamidine If anaerobe: cef and met

TB Skin
2mo: RHZE Impetigo
4mo: RH Localised: topical fusidate
Give pyridoxine 20mg OD throughout Rx Widespread: fluclox 250 QDS
Longer Rx if resistant organisms or extra-pulmonary TB Erysipelas: Pen V 500mg QDS or ben pen
Cellulitis
Empiric: fluclox 500mg QDS
Infective Endocarditis Known Strep: Pen V or Ben pen
Empiric
Acute severe: Fuclox + gent IV
Subacute: Benpen + gent IV

Streps: benpen + gent IV


Enterococci: amoxicillin + gent IV
Staphs: fluclox rifampicin IV
Fungi: flucytosine IV + fluconazole PO.
Amphotericin if flucytosine resistance or
Aspergillus.

Alasdair Scott, 2012 58


Viral Infections
Herpes Viruses Influenza
Bed rest + paracetamol
HSV If severe
Aciclovir Mx in ITU
Valaciclovir Cipro and ao-amoxiclav: prevent Staph and Strep
Famciclovir Oseltamivir
Neuraminidase inhibitor active vs. flu A and B
VZV May be indicated if >1yr c symptoms of <48hr
Aciclovir Zanamivir
Inhaled NA inhibitor active vs. influenza A and B
CMV >5yrs
c symptoms <48h
Ganciclovir / valganciclovir
Foscarnet
Cidofovir HIV
Indications
Hepatitis CD4 350
AIDS-defining illness
HBV Pregnancy
Combination or individual use of HIVAN
Peg-interferon- Co-infected
c HBV when Rx is indicated for HBV
Tenofovir
Entecavir Regimens
1 NNRTI + 2 NRTIs
HCV NNRTI = efavirenz
Peg-interferon- + ribavarin NRTI = emtricitabine + tenofovir (Truvada)
Atripla = efavirenz + emtricitabine + tenofovir
PI + 2 NRTIs
PI = lopinavir (+ low dose ritonavir = Kaletra)

Aim
Undetectable VL after 4mo
If VL remains high despite good compliance
Change to a new drug combination
Request resistance studies

Alasdair Scott, 2012 59


Protozoal Infections Fungal Infections
Malaria PCP
High-dose co-trimoxazole IV
Prophylaxis Or, pentamidine IVI
No resistance: proguanil + chloroquine Prednisolone if severe hypoxaemia
Resistance: mefloquine or malarone
Malarone: atovaquone + proguanil
Cryptococcal Meningitis
Rx Amphotericin B + flucytosine for 2wks
Benign: chloroquine then primaquine Then fluconazole for for 6mo / until CD4 >200
Falciparum: riamet or malarone
Riamet: artemether + lumefantrine
IV antimalarials if severe
Candida
Oral: nystatin drops
Toxoplasmaosis Thrush: clotrimazole
Pyrimethamine + sulfadiazine Systemic: amphotericin B or voriconazole

Giardia Tinea Infections


Tinidazole Skin: terbinafine or topical ketoconazole / miconazole
Scalp: griseofulvin or terbinafine
Nails: terbinafine
Entamaeoba histolytica
Metronidazole
Pityriasis versicolor
Selenium or ketoconazole shampoo

Aspergillus
Amphotercin B
Itraconazole, voriconazole

Alasdair Scott, 2012 60


Endocrine
Contents
Oral Hypoglycaemics................................................................................................................................................................... 62
Insulin .......................................................................................................................................................................................... 63
Thyroid Drugs .............................................................................................................................................................................. 64
Pituitary Drugs ............................................................................................................................................................................. 64
Calcium Metabolism .................................................................................................................................................................... 65
Sex Hormones ............................................................................................................................................................................. 66
T2DM ........................................................................................................................................................................................... 67
Insulin Dependent DM ................................................................................................................................................................. 67
Thyroid Disorders ........................................................................................................................................................................ 68
Thyrotoxicosis.............................................................................................................................................................................. 68
Adrenal Steroids .......................................................................................................................................................................... 68
Pituitary Disorders ....................................................................................................................................................................... 69
Osteoporosis ............................................................................................................................................................................... 69

Alasdair Scott, 2012 61


Oral Hypoglycaemics
Drug MOA Side Effects Contraindications Interactions Other
Metformin Biguanide Lactic acidosis Caution in renal or Renally excreted
Insulin sensitizer hepatic impairment - dose or avoid if eGFR
- gluconeogenesis GI upset
- peripheral glucose use - anorexia wt. loss Contrast media Cannot cause hypos
- LDL and VLDL General anaesthesia
Recent MI
Pioglitazone Thiazolidinedione Wt. gain H/L Dont use with insulin
Peripheral insulin sensitizer Fluid retention V. protein bound
PPAR gamma ligand (nuclear receptor Hepatotoxicity Insulin use Hepatic metabolism
involved in glucose and lipid homeostasis) May exacerbate HF ACS
Monitor LFTs
Gliclazide (s) Sulfonylureas Hypoglycaemia Severe L/R fx d by: Renally excreted
Tolbutamide (s) Insulin secretagogues - can be prolonged - sulphonamides
Glipizide (s) Acute porphyria - trimethoprim V. albumin bound
Glibenclamide (l) Block hyperpolarising K+ channel on cells Wt. gain ( appetite) - NSAIDs
depolarisation and insulin release GI upset - warfarin Caution in elderly
c renal function
(Chlorpropamide) Headache - fibrates
Avoid long-acting (glibenclamide) in
elderly use gliclazide or tolbutamide
Nateglinide Meglitinides Hypoglycaemia V. short acting risk of hypo
Repaglinide Insulin secretagogues Give before meal
Block hyperpolarising K+ channel
Exenatide Insulin secretagogue Hypoglycaemia Admin by SC injection
Liraglutide GLP-1 analogue GI upset
- insulin secretion and sensitisation
Sitagliptin Insulin secretagogues Hypoglycaemia
Vildagliptin Dipeptidylpeptidase-4 inhibitor GI upset
DPP-4 breaks down endogenous GLP-1
Acarbose Intestinal -glucosidase inhibitor Flatulence IBD Monitor LFTs
Delays carb absorption post-prandial Loose stools/diarrhoea L
blood glucose Abdo pain / bloating
Little effect on fasting glucose Hepatotoxicity (rare)

Alasdair Scott, 2012 62


Insulin
Short-acting Intermediate- and long- acting
Examples Soluble Insulin Insulin Analogues Isophane Insulin Insulin Analogues
Actrapid Aspart: NovoRapid Insulatard Glargine: Lantus
Humulin S Lispro: Humalog Humulin I Detemir: Levemir
Onset (sc) 30-60min 15min 1-3hrs
Peak (sc) 2-4hrs 1-3hrs 4-12hrs
Duration (sc) 6-8hrs 2-5hrs 12hrs Glargine: 24hrs
Detemir: 20hrs
Uses Maintenance: 15-30min before meals Glargine is given OD (nocte) as basal therapy
DKA Long acting analogues nocturnal hypoglycaemia
Sliding scales

Effects of insulin Side Effects


Hypoglycaemia
Adipose tissue At risk: EtOH binge, -B (mask symptoms),
lipoprotein lipase activity TGs elderly
GLUT-4 activity glucose storage as fat Need to admit sulfonylurea-induced hypo
lipolysis fatty acids release into circulation Lipohypertrophy
Rotate injection site: abdomen, thighs
Liver Wt. gain in T2DM
glycogenolysis wt. gain if insulin given
c metformin
gluconeogenesis
glycogenesis Problems c Actrapid in the Basal-Bolus Regime
Inhibition of ketogenesis
Absorbed over 3-4hrs: not physiological
Must give 15min before meal
Muscle
Can late post prandial hypoglycaemia
proteolysis
Immediate post-prandial hyperglycaemia may risk
GLUT-4 activity glucose uptake
of DM complications
NB. Lipoprotein lipase allows triglyceride uptake from LDL by
adipocytes.
Short Acting Insulin Analogues
Fast onset and shorter duration
Factors Affecting Absorption Can be given just before start of meal
Temperature Less chance of post-prandial hypoglycaemia
Exercise
Preparation: actrapid vs. glargine
Dlvry method
IV: 1min peak conc
SC: 90min peak conc
Injection site and depth

Administration
s/c: typical route
IVI
DKA
Control in critical illness
Control in peri-operative period

Alasdair Scott, 2012 63


Thyroid Drugs

Drug MOA Side Effects Contraindications Interactions Other


Thyroxine Precipitation of heart failure W+ Introduce T4 slowly in the elderly
Osteopaenia
AF
Over-Rx hyperthyroidism
Carbimazole Thionamides Agranulocytosis Pregnancy Propythiouracil is reserved for those
Propythiouracil - thyroperoxidase inhibitors - often transient and benign Children intoleant of carbimazole due to risk of
- prevent iodination of tyrosine Tracheal compression hepatitis
- T4/T3 synthesis Hypersensitvity: rash, pruritis - do surgery
Titrate to normal TSH or block and
Carbimazole is a pro-drug Hepatitis replace.
- converted to methimazole
Radioiodine: I131 Radioiodine is localised to thyroid hypothyroidism Stop thionamide before use
destruction of gland Can precipitate thyroid storm

Pituitary Drugs

Drug MOA Side Effects Contraindications Interactions Other


Bromocriptine Ergot-derived Da agonists Fibrosis Cardiovascular disease Levels d by: Use
Cabergoline GI upset: esp. nausea Porphyria - octreotide - prolactinoma
Pergolide Postural hypotension Psychosis - macrolides - can be used in acromegaly
Drowsiness
Neuropsych syndromes Monitor heart c echo
Octreotide Somatostatin analogues Diarrhoea Use
Lanreotide Gallstones - acromegaly
- carcinoid syndrome
Pegvisomant GH receptor antagonist Use
- acromegaly
Metyrapone 11-hydroxylase inhibitor Hypoadrenalism Use
- inhibits adrenal cortisol - can be used Cushings syn.
production - particularly if resistant to surgery

Alasdair Scott, 2012 64


Calcium Metabolism

Drug MOA Side Effects Contraindications Interactions Other


Cinacalcet Calcimimetic Used for Rx of 2O HPT in ESRF
- PTH secretion
Sevelamer Phosphate binder GI upset GI obstruction Used to PO4 in ESRF
Lanthanum
Alendronate Bisphosphonates GI upset Achalasia Use
Etidronate ostoclastic bone resorption Oseophagitis and ulcers Oesophageal stricture - Prevention of osteoporotic #s
Pamidronate Osteonecrosis of the jaw - Prevention of osteoporosis
Ibandronate Diffuse MSK pain - Hypercalcaemia of malignancy
Risedronate - Pagets
Zoledronate
Take c glass of water on an empty
stomach 30min before breakfast,
staying upright.
Strontium bone formation DRESS syndrome Use if bisphosphonates not tolerated
bone resorption - Drug Rash
- Eosinophilia
- Systemic Symptoms: fever, wcc
Teriparetide Recombinant PTH GI upset Skeletal malignancies
- pulsatile admin bone Pagets
formation and resorption Severe R
Denosumab Anti-RANK ligand Used if bisphosphonates not tolerated
- osteoclast activation
Ergocalciferol Vitamin D2 Commonly used for Vit D deficiency
Colecalciferol Vitamin D3
Alfacalcidol 1 (OH) Vit D3 Use in renal disease
Calcitriol 1, 25, (OH) Vit D3 Use in renal disease

Alasdair Scott, 2012 65


Sex Hormones

Drug Benefits Side Effects Contraindications Interactions Other


COCP Contraception risk of VTE Personal Hx of VTE P450 metabolism Dont need extra
dysmenorrhoea, menorrhagia Small risk of breast Ca Risk of VTE - effectiveness
c contraception when taking c
PMT Small risk of IHD Risk of arterial disease enzyme inducers oral Abx that dont induce
benign breast disease Gallstones Hx of breast Ca liver enzymes, unless d/v
ovarian and endometrial Ca Cholestatic jaundice fx of steroids
risk of PID Breast tenderness
Hepatoma
POP n/v Severe arterial disease P450 metabolism
Headache Hx of breast Ca - effectiveness
c
wt. enzyme inducers
Breast tenderness
HRT hot flushes, vaginal dryness, libido risk of Ca: breast, endometrial, ovarian Oestrogen-dependent Ca Excess Ca risk disappears w/i
urinary frequency / urgency risk of VTE Hx of breast Ca 5yrs of stopping
risk of bowel Ca risk of stroke and IHD UnDx vaginal bleeding
risk of osteoporotic #s Cholestatic jaundice VTE

COCP Cautions and Contraindications


VTE Risk: caution if 1, avoid if 2
FHx of VTE
BMI >30, avoid if >35
Long-term immobilisation
Hx of superficial thrombophlebitis
>35yrs, avoid if >50yrs
Smoking

Arterial Risk: caution if 1, avoid if 2


FHx of arterial disease
DM
HTN
Smoking, avoid if >40/d
>35yrs, avoid if >50yrs
Migraine w/o aura, avoid if migraine
c aura

Alasdair Scott, 2012 66


T2DM Insulin Dependent DM
1. Lifestyle Modification: DELAYS Principles
Diet Ensure pt. education about
Exercise Self-adjustment c exercise and calories
Lipids (statins if >40 regardless of lipids) Titrate dose
Aspirin (consider if >50yrs or <50 c other RFs) Family member can abort hypo c sugary drinks
ABP or GlucoGel
Yearly / 6moly f/up Pre-prandial BM dont tell you how much glucose is
Smoking cessation needed
Fasting BM before meal informs re long-acting insulin
dose.
Finger-prick BM after meal informs re short-acting
2. Start Metformin
insulin dose (for that last meal)
(if HBA1c >target after lifestyle changes)
SE: nausea, diarrhoea, abdo pain, lactic acidosis
CI: GFR<30, tissue hypoxia (sepsis, MI), morning
before GA and iodinated contrast media
Common Regimes
500mg after evening meal, ing to 2g max.
BD Biphasic Regime
BD insulin mixture 30min before breakfast and dinner
Rapid-acting: e.g. actrapid
3. Metformin + Sulfonylurea Intermediate- / long-acting: e.g. insulatard
(if HBA1c >target)
T2 or T1 DM c regular lifestyle: children, older pts.
E.g. gliclazide MR 30mg c breakfast
Assoc, c fasting hyperglycaemia
SE: hypoglycaemia, wt. gain
CI: omit on morning of surgery Basal-Bolus Regime
Bedtime long-acting (e.g. glargine) + short acting before
Other Options
each meal (e.g. lispro)
Consider adding a rapid-acting insulin Adjust dose according to meal size
secretagogue (e.g. nateglinide) to metformin ~50% of insulin given as long-acting
instead of a sulfonylurea.
T1DM allowing flexible lifestyle
May be preferable if erratic lifestyle.
Best outcome
Consider adding pioglitazone to metformin
instead of a sulfonylurea
OD Long-Acting Before Bed
Initial regime when switching from tablets in T2DM
4. Additional Therapy
Illness
st
1 line Insulin requirements usually (even if food intake )
Add insulin insulin + metformin + sulfonylurea Maintain calories (e.g. milk)
Check BMs 4hrly and test for ketonuria
2nd line insulin dose if glucose rising
Add sitagliptin or pioglitazone if insulin
unacceptable
Employment, social or recreational issues
Obesity
Metformin + sulfonylurea + sitagliptin / pio

3rd line
Add exenatide (SC) if insulin unacceptable or
BMI>35
Metformin + sulfonylurea + exenatide

4th line
Consider acarbose if unable to use other glucose-
lowering drugs

Targets
Capillary blood glucose
Fasting: 4.5-6.5mM
2h post-prandial: 4.5-9mM
HbA1c
Reflects exposure over last 6-8wks
Aim <45 - 50mM (6 - 7%)

Alasdair Scott, 2012 67


Thyroid Disorders Adrenal Steroids
Hypothyroidism Glucocorticoid Replacement
Need for 1O and 2O adrenal failure
Levothyroxine
Titrate to normalize TSH Hydrocortisone
Enzyme inducers thyroxine metabolism Preferred due to gluco- and mineralo-corticoid activity.
Clinical improvement takes ~2wks 20-30mg in divided doses
10mg AM, 5mg lunch, 5mg evening
Caution
In elderly pts.
c subclinical heart failure thyroxine can dose at intercurrent illness / injury
ppt. acute worsening: palpitations, angina, MI If eating
Introduce thyroxine gradually 3x normal dose for 3d
Excessive thyroxine osteopaenia and AF Then 2x normal dose for 3d
Vomiting / cant tolerate PO
100mg IV TDS

Thyrotoxicosis
Cushings
Confirm Aetiology Treat underlying cause: e.g. pituitary or adrenal tumour
Use Tc scan to determine if thyroxicosis is high or low May use drugs temporarily or permanently if pt. cant
uptake. undergo surgery: e.g. lung Ca
Low
Subacute de Quervains thyroiditis Metyrapone
Postpartum thyroiditis Inhibits final step in cortisol synthesis
Amiodarone 80% response in Cushings disease
High Usually temporary
Graves: 40-60% Can be used as part of block and replace strategy
TMNG: 30-50%
Thyroid Adenoma: 5% Other anti-glucocorticoid drugs
Functioning thyroid Ca Ketoconazole: inhibits steroid synthesis
Mifepristone: progesterone and glucocorticoid receptor
Mx antagonist
Low Uptake
Symptomatic: propranolol, atenolol Mineralocorticoid Replacement
NSAIDs for de Quervains
Need for 1O adrenal failure only
High Uptake
-B Fludrocortisone
Carbimazole
Balance between HTN and postural hypotension
Titration to normal TSH
Or, block and replace
Radioiodine
Surgery
Primary Hyperaldosteronism
Bilateral adrenal hyperplasia (70%)
Spironolactone
Eplerenone
Amiloride

Conns adenoma (30%)


Surgical excision or medical therapy

Alasdair Scott, 2012 68


Pituitary Disorders Osteoporosis
Cranial DI Conservative
Desmopressin Stop smoking, EtOH
Nasal spray Wt. bearing or balancing exercise (e.g. Tai Chi)
IV Ca and vit-D rich diet
Avoid dilutional hyponatraemia by daily polyuria Home-based fall-prevention program c visual
episode assessment.

Acromegaly 1O and 2O prevention of osteoporotic #s


st
1 line: trans-sphenoidal excision Bisphosphonates: alendronate is 1st line
2nd line: somatostatin analogues octreotide Ca and Vit D supplements
3rd line: GH antagonist pegvisomant e.g Calcium D3 Forte
4th line: radiotherapy Strontium ranelate: bisphosphonate alternative

Somatostatin Analogues
90% respond Alternative for 2O prevention of osteoporotic #s
IGF1 normalised in 60% Teriparetide: PTH analogue new bone formation
Denosumab: anti-RANKL osteoclast activation
Raloxifene: SERM, breast Ca risk cf. HRT
Prolactinoma
1st line: D2 agonist
2nd line: Trans-sphenoidal excision
If visual or pressure symptoms dont response to
medical Rx

D2 Agonists
Bromocroptine, Cabergoline
PRL secretion and tumour size

Hypopituitarism
ACTH: hydrocortisone
GH: rh-GH
FSH / LH
Testosterone
OCP
TSH: T4

Alasdair Scott, 2012 69


Malignancy and
Immunosuppression
Contents
Anti-Proliferative Agents .............................................................................................................................................................. 71
Inhibitors of Cell Signalling .......................................................................................................................................................... 71
Chemotherapeutics ..................................................................................................................................................................... 72
Immunosuppression .................................................................................................................................................................... 73

Alasdair Scott, 2012 70


Anti-Proliferative Agents

Drug MOA Side Effects Contraindications Interactions Use Other


Cyclophosphamide Alkylates DNA BM suppression Cancer Give mesna to prevent
Affects B cells > T cells Haemorrhagic cystits RA haemorrhagic cystitis
Alopecia SLE
Sterility: esp. males Systemic sclerosis Activated by P450
Wegeners

Cisplatin Alkylates DNA BM suppression Cancer Carboplatin is assoc.


c
Severe n/v - Ovarian less severe SEs
Nephrotoxic - Lung: SSLC
Ototoxic - Testicular Requires pre-admin
Peripheral neuropathy hydration
Azathioprine Blocks de novo purine BM suppression Allopurinol toxicity Prevent Tx rejection Do TPMT (thiopurine
synthesis Hepatotoxicity Steroid-sparing agent methyltransferase) assay
Active metabolite is 6- n/v/d - IBD before use.
mercaptopurine Arthralgia - SLE
Affects T cells > B cells - RA 50% of pts intolerant of
azathioprine tolerate 6-MP
Mycophenolate Blocks de novo nucleotide BM suppression Prevent Tx rejection
mofetil synthesis Skin malignancy AI disease
Affects T cells > B cells GI upset

Methotrexate Dihydrofolate reductase BM suppression R toxicity


c Cancer Give c folinic acid to
inhibitor Pulmonary fibrosis L - NSAIDs RA risk of myelosuppression
Hepatotoxic - ciclosporin Psoriasis
Mucositis - steroids Crohns Monitor U+E, FBC, LFT
Chlorambucil Alkylates DNA BM suppression Cancer: e.g. CLL
EM SJS

Inhibitors of Cell Signalling

Drug MOA Side Effects Contraindications Interactions Use Other


Ciclosporin Calcineurin inhibitors Nephrotoxic P450 metabolism Prevent Tx rejection Monitor LFTs
- Blocks IL-2 production Hepatic dysfunction GvHD
Tremor UC
Hypertrichosis RA
Gingival hypertrophy Psoriasis
Encephalopathy
Tacrolimus Nephrotoxic (< cf. ciclosporin) P450 metabolism Prevent Tx rejection
Diabetogenic
Neurotoxic (> cf. ciclosporin)
Sirolimus Blocks mTOR pathway Dyslipidaemia Prevent Tx rejection
Alasdair Scott, 2012 71
Chemotherapeutics
Important Cytotoxic Classes Specific Problems

Alkylating agents Drug Problems


Cyclophosphamide, chlorambucil, busulfan, cisplatin Cyclophosphamide Haemorrhagic cystitis: give mesna
Mechanism Hair loss
DNA x-linking BM suppression
Base mis-paring Doxorubicin + other Cardiomyopathy
Excision of alkylated DNA strand breaks anthracyclines Extravasation reactions
Bleomycin Pulmonary fibrosis
Antimetabolites: methotrexate, 5-FU Vincristine Peripheral neuropathy
- dont give IT
Cytotoxic Abx Cisplatin Severe n/v
Anthracyclines: doxorubicin, daunorubicin Nephrotoxic
Bleomycin Ototoxic
Mechanism Peripheral neuropathy
Intercalate
c DNA Paclitaxel Peripheral neuropathy
Free radical formation Hypersensitivity
- Pre-Rx c antihistamines + steroids
Microtubule Inhibitors 5-FU Palmar-plantar erythrodysthesia
Vinca alkaloids: vincristine, vinblastine Mucositis
Taxanes: paclitaxel

Topoisomerase Inhibitors Mx Chemo-induce Emesis


Etoposide
Low risk of emesis
Domperidone / metoclopramide started pre-Rx
Immune Modulators: thalidomide, lenalidomide
High risk of emesis
MAbs Ondansetron / granisetron +
Trastuzumab (anti-Her2): breast Ca Dexamthasone +
Bevacizumab (anti-VEGF): RCC, CRC, lung Aprepitant
Cetuximab (anti-EGFR): CRC
Rituximab (anti-CD20): NHL Common Regimens
Tyrosine Kinase Inhibitors Breast Ca: FEC
Erlotinib: lung Ca 5-FU
Imatinib: CML Epirubicin
Sunitinib: RCC Cyclophosphamide
Endocrine Modulators: tamoxifen, anastrazole Breast Ca: CMF
Cyclophosphamide
Methotrexate
Common Side Effects 5-FU
n/v: prophylactic anti-emetics
Alopecia Testicular Teratoma: BEP
Neutropenia: 10-14d after chemo Bleomycin
Extravasation of chemo agent Etoposide
Pain, burring, bruising @ infusion site CisPlatin
Stop infusion, give steroids, apply cold pack
Liaise early c plastics Ovarian
Hyperuricaemia Carboplatin
Oral mucositis Paclitaxel

NHL: R-CHOP
Rituximab
Cyclophosphamide
Hydroxydaunomycin (doxorubicin)
Oncovin
Prednisolone

HL: ABVD
Adriamycin (doxorubicin)
Bleomycin
Vinblastine
Dacarbazine
Alasdair Scott, 2012 72
Immunosuppression
Prednisolone Transplant Regimens
MOA Liver
Inhibits PLA2 PG and PAF Tacrolimus
PMN extravasation PMN in blood Azathioprine
Lymphocyte sequestration in tissues lymphopenia Prednisolone withdrawal @ 3mo
Phagocytosis
Lymphocyte apoptosis Renal
Ab production Pre-op induction
cytokine and proteolytic enzyme release Alemtuzumab (Campath: anti-CD52)
Post-op
Dose Predniolone 7d
Use lowest possible dose: alternate days if possible Tacrolimus long-term
Graded withdrawal if used >3wks

Advice
Dont stop steroids suddenly
Consult doctor when unwell
dose
c illness or stress (e.g. pre-op)
Carry a steroid card: dose and indication
Avoid OTCs: e.g. NSAIDs
Osteoporosis and PUD prophylaxis
Ca and vitamin D supplements: Calcichew-D3
Bisphosphonates: alendronate
PPI: lansoprazole

SEs (mostly long-term use >6wks)

GI Candidiasis
PUD
Oesophageal ulceration
Pancreatitis

Cardio HTN
CCF

MSK Proximal myopathy


Osteoporosis

Endo Growth suppression


HPA suppression
Cushings syndrome

Metabolic Na and fluid retention


PMN
K

CNS Depression, psychosis

Eye Cataracts
Glaucoma

Immune susceptibility to infection

Interactions
Fx d by hepatic inducers
Fx d by
Hepatic inhibitors
OCP

Alasdair Scott, 2012 73


Musculoskeletal
Contents
Rheumatic Disease ..................................................................................................................................................................... 75
Anti-Gout ..................................................................................................................................................................................... 75
NSAIDs ........................................................................................................................................................................................ 76
Neuromuscular Drugs ................................................................................................................................................................. 77
Osteoarthritis ............................................................................................................................................................................... 78
Rheumatoid Arthritis .................................................................................................................................................................... 78
Gout ............................................................................................................................................................................................. 79

Alasdair Scott, 2012 74


Rheumatic Disease

Drug MOA Side Effects Contraindications Interactions Other


Sulfasalazine 5-Aminosalicylate Sulfasalazine has SEs Caution in renal or Monitor FBC
Mesalazine - blood dyscrasias hepatic impairment
Unknown MOA - hepatitis Better for use in young women
- rash, urticaria cf. methotrexate
- oligospermia
- pulmonary fibrosis
Methotrexate Dihydrofolate reductase inhibitor BM suppression R toxicity
c Cancer
Pulmonary fibrosis L - NSAIDs RA
Hepatotoxic - ciclosporin Psoriasis
- steroids Crohns
Hydroxychloroquine activation of dendritic cells Visual change Caution in G6PD Monitor vision
- rarely retinopathy deficiency
Seizures
BM suppression
Penicillamine Nephrotic syndrome SLE Chelates Cu and Pb
Drug-induced lupus Prevents stones in cystinuria
Taste change
Infliximab Chimeric anti-TNF mAb Severe infections TB Screen for TB before use
- Remicade TB
Allergic reactions Parenteral admin
CCF
CNS demyelination Give c hydrocortisone to
AI disease and C allergic SEs

Anti-Gout

Drug MOA Side Effects Contraindications Interactions Other


Colchicine Diarrhoea Caution in renal impairment
Renal impairment
Allopurinol XO inhibitor Severe skin reactions Caution in R and L metabolism of azathioprine Initial Rx can gout
- EM SJS - dose - AVOID - initiate
c NSAID / coplchicine cover
GI upset
Hepatotoxic
Feboxustat XO inhibitor Headache Caution in R and L
Rash
Abnormal LFTs
Probenecid Uricosuric GI upset Renal impairment
Rasburicase Recombinant uric
oxidase
Alasdair Scott, 2012 75
NSAIDs

Drug MOA Side Effects Contraindications Interactions Other


Least Most Toxic Non-selective COX inhibitors Gastritis and PUD Renal or cardiac failure bleeding c warfarin Can be given c other agents
- Ibuprofen GFR PUD s effects of ACEis and ARBs for gastro protection
- Diclofenac Analgesic Interstitial nephritis Severe hepatic impairment toxicity of methotrexate - PPI
- Aspirin Antipyretic Papillary necrosis - H2RAs
- Naproxen Anti-inflammatory K Caution - mioprostol
- Indomethacin Peripheral oedema - in the elderly
- Ketoprofen Bronchospasm - asthma
Hypersensitivity
- EM SJS
Celecoxib Selective COX-2 inhibitor cardiovascular IHD s effects of ACEis and ARBs Assess CV risk before use
events Cerebrovascular disease toxicity of methotrexate
Renal SEs as above L/R failure Only used for short periods
in young pts.
c intolerance
of other NSAIDs

Eicosanoid Synthesis

Alasdair Scott, 2012 76


Neuromuscular Drugs

Drug MOA Side Effects Contraindications Interactions Other


Pyridostigmine Anticholinesterases Cholinergic Asthma Edrophonium preferred for the Dx of MG
Neostigmine - ACh in the synaptic cleft Intestinal / urinary obstruction due to its v. short t.
Edrophonium - Enhance neuromuscular transmission
Pyridostigmine preferred for the Rx of
MG due to long t
Baclofen GABAB agonist Sedation PUD fx d by TCAs Rx painful muscle spasms
tone
Skeletal muscle relaxant Nausea Dont withdraw abruptly
Urinary disturbance - hyperthermia
- spasticity
Dantrolene Prevents Ca2+ release from Hepatotoxicity Hepatic impairment Used to relieve chronic spasticity and in
sarcoplasmic reticulum GI upset malignant hyperthermia

Skeletal muscle relaxant

Alasdair Scott, 2012 77


Osteoarthritis Rheumatoid Arthritis
Conservative Conservative
wt. Refer to rheumatologist
Alter activities: rest, sport Regular exercise
Physio: muscle strengthening PT
Walking aids, supportive footwear, home mods OT: aids, splints

Medical Medical
Analgesia DAS28: monitor disease activity
Paracetamol DMARDs and biologicals: use early
NSAIDs Steroids: PO or intra-articular for exacerbations
1st: Ibuprofen: 400mg TDS NSAIDs: good for symptom relief
2nd: diclofenac e.g. arthrotec: but diarrhoea
Tramol Mx CV risk: RA accelerates atherosclerosis
Joint injection: local anaesthetic and steroids Prevent osteoporosis and gastric ulcers

Surgical DMARDs
Arthroscopic washout: esp. knee.
1st line for treating RA
Trim cartilage, remove foreign bodies.
Early DMARD use assoc. c better long-term outcome
Arthroplasty: Replacement (or excision)
Osteotomy: small area of bone cut out. All DMARDs can myelosuppression pancytopenia
Arthrodesis: last resort for pain management
Main agents:
Novel Techniques
Methotrexate: hepatotoxic, pulm. fibrosis, teratogenic
Microfracture: stem cell release fibro-cartilage
formation Sulfasalazine: SJS, sperm count, pulmonary fibrosis
Autologous chondrocyte implantation Hydroxychloroquine: visual change, rash, seizures

Other Agents:
Leflunomide: risk of infection and malignancy
Gold: nephrotic syndrome
Penicillamine: drug-induced lupus, taste change,
nephrotic syn.

Biologicals
Anti-TNF
Severe RA not responding to DMARDs
DAS28 >5.1
Failed methotrexate + 1 other DMARD
Screen and Rx TB first
Infliximab: chimeric anti-TNF Ab
Etanercept: TNF-receptor
Adalimumab: human anti-TNF Ab
SEs: infection (sepsis, TB), AI disease, Ca

Rituximab (anti-CD20 mAb)


Severe RA not responding to anti-TNF therapy

Surgical
Ulna stylectomy
Joint prosthesis

Alasdair Scott, 2012 78


Gout
Acute Rx
NSAID: diclofenac or indomethacin
Colchicine
NSAIDs CI: warfarin, ulcers, heart HF, CRF
SE: diarrhoea
In renal impairment: NSAIDs and colchicine are CI
Use steroids

Prevention
Conservative
Lose wt.
Avoid prolonged fasts and EtOH excess
Xanthine Oxidase Inhibitors: Allopurinol
Use if recurrent attacks, tophi or renal stones
Introduce
c NSAID or colchicine cover for 3/12
SE: rash, fever, WCC (c azathioprine)
Use febuxostat (XO inhibitor) if hypersensitivity
Uricosuric drugs: e.g. probenecid, losartan
Rarely used
Recombinant urate oxidase: rasburicase
May be used pre-cytotoxic therapy

Alasdair Scott, 2012 79


Emergencies
Contents
Cardiovascular............................................................................................................................................................................. 81
Neurological................................................................................................................................................................................. 82
Respiratory .................................................................................................................................................................................. 82
Endocrine .................................................................................................................................................................................... 83
Metabolic ..................................................................................................................................................................................... 84

Alasdair Scott, 2012 80


Cardiovascular
ACS Malignant HTN
Oxygen
Analgesia BP > 220 / 120 + either
Morphine 5-10mg IV Grade 3: haemorrhages, exudates
Metoclopramide 10mg IV Grade 4: papilloedema
Anti-platelet
Aspirin 300mg PO Mx
Clopidogrel 300mg PO Aim to DBP to 100-115mmHg over 4-6hrs
Anti-ischaemia Nitroprusside or labetalol
GTN: 2 puffs or 1 tab SL Phentolamine if phaeo known or suspected
IV if pain continues
Atenolol SVT
STEMI: 5mg IV
NSTEMI: 50mg PO /24h Unstable DC cardioversion
Anti-coagulate
Enoxaparin Irregular Rhythm: Rx as AF

STEMI Regular Rhythm


st
1 : PCI Adenosine 6mg IV bolus, then 12mg, then 12mg
2nd: thrombolysis Choose from
Add tirofiban in high-risk pts. Verapamil
Atenolol
NSTEMI Digoxin
Tirofiban Amiodarone
Angiography PCI w/i 96hrs
Broad Complex Tachycardia
Long-Term
ACEi: e.g. lisinopril 2.5mg OD Unstable DC cardioversion
Aspirin 75mg OD
-B: e.g. bisoprolol 1.25mg OD Regular Rhythm
Verapamil if -B not tolerated Amiodarone or lignocaine
Clopidogrel 75mg OD
STEMI: 1mo Irregular Rhythm
NSTEMI: 1yr Pre-excited AF: amiodarone or flecainide
Statin: e.g. atorvastatin 80mg OD TDP: MgSO4

Septic Shock
Acute Pulmonary Oedema
Initial Rx
Initial Mx Oxygen
Oxygen Abx: cultures 1st then follow guidelines (e.g Tazocin)
Diamorphine 2.5-5mg + metoclopramide Fluids: 1L crystalloid or 500ml colloid over 30min
Frusemide 40-80mg IV If BP still consider CVP and further fluids
GTN 2 puffs or 2 tabs SL Aim CVP 8mmHg and UO >0.5ml/kg/hr
ISMN 2-10mg/h IVI Inotropes if SBP <90mmHg after fluid resus
Keep SBP >90mmHg Norad 1-10ug/min: maintain MAP >65mmHg

Rx underlying cause Further Mx


Insulin sliding scale: keep glucose <8.3mM
If no improvement DVT prophylaxis: enoxaparin 40mg OD SC
CPAP Stress ulcer prophylaxis: PPI
If SBP <90: dobutamine
2-20ug/kg/min IVI Cardiac Arrest
Via a central line Shockable
Adrenaline 1mg + amiodarone 300mg after 3rd
Anaphylaxis shock
Oxygen Repeat adrenaline every other cycle
Adrenaline 0.5mg IM: 0.5ml of 1:1000 Non-Shockable
Repeat every 5min as necessary, guided by Adrenaline 1mg as soon as IV access obtained
cardiorespiratory function Repeat adrenaline every other cycle
Chlorphenamine 10mg IV
Hydrocortisone 200mg IV Bradycardia <40bpm
Nebs: salbutamol 5mg + ipratropium 0.5mg Atropine 0.61.2g (max 3g) IV
Isoprenaline IVI

Alasdair Scott, 2012 81


Neurological Respiratory
Meningitis Acute Asthma
Acute Mx Initial Rx
Septicaemic O2 driven nebulisers
Ceftriaxone 2g IVI Salbutamol 5mg
+ ampicillin 2g IVI /4h if >50yrs Ipratropium 0.5mg
Meningitic Repeat salbutamol every 15min until
Ceftriaxone ampicillin post-LP improvement
Dexamethasone 0.15mg/kg IVI QDS Monitor ECG
Steroids
Continuing Mx 100mg hydrocortisone
Ceftriaxone 2g IVI BD Or, 50mg prednisolone

No Improvement
Status Epilepticus MgSO4 2g IVI over 20min
Salbutamol 3-20 ug/min IVI
Reverse Potential Causes Aminophylline
Thiamine 250mg IV if EtOH If not already on theophylline
100ml 20% glucose unless known to be normal Load then IVI

IV Bolus Phase
Lorazepam 2-4mg IV over 30s Acute COPD
Repeat if no response w/i 2min
Alternatives Initial Rx
Diazepam 10mg IV/PR (20mg max) 24% O2
Midazolam 10mg buccal Blue venturi mask
Aim for 88-92% SpO2
IV Infusion Phase Hydrocortisone 200mg IV
Phenytoin 18mg/kg IVI Doxy 200mg PO STAT if evidence of infection
50mg/min max
Or, diazepam IVI No Improvement
Consider aminophylline
Induction Phase BiPAP
Propofol or thiopentone

PE
Oxygen
Morphine + metoclopramide
Massive PE: alteplase 50mg bolus STAT
Stable: anticoagulate
Enoxaparin 1.5mg/kg/24hr SC
Warfarin: 5mg PO

Alasdair Scott, 2012 82


Endocrine
DKA Thyroid Storm
Fluid resuscitation + NGT
Dx Bloods: TFTs + cultures if infection suspected
Acidosis (AG): pH <7.3 ( HCO3 <15mM) Propranolol PO/IV
Hyperglycaemia: 11.1mM (or known DM) Digoxin + LMWH if AF
Ketonaemia: 3mM (2+ on dipstix) Carbimazole then Lugols Iodine 4h later to inhibit
thyroid
Fluids Hydrocortisone 100mg QDS IV
0.9% NS infusion via large-bore cannula Rx cause
SBP<90 1L stat + more until SBP >90
SBP>90 1L over 1h
Then: 1L over next 2h, 1L/2h, 1L/4h, 1L/4h, 1L/6h
Myxoedema Coma
Switch to 10% dex 1L/8h when glucose <14mM
Correct any hypoglycaemia
T3/T4 IV slowly (may ppt. myocardial ischaemia)
Insulin Infusion
0.1u/kg/h Actrapid (6u if no wt., max 15u) Hydrocortisone 100mg IV
Transfer to sliding scale when resolved Rx hypothermia and heart failure
Ketones <0.3mM
Venous pH >7.3 (HCO3 >18mM)
Transfer to SC insulin when eating Addisonian Crisis
Check CBG: glucose may be needed
nd
Start Potassium Replacement in 2 Bag of Fluids Hydrocortisone 100mg IV 6hrly
>5.5mM Nil IV crystalloid
3.5-5.5mM 40mM Rx underlying cause
<3.5mM consult senior for review

Additional Therapy Phaeochromocytoma Emergencies


LMWH 1st -block
Consider NaHCO3 if pH <6.9 Phentolamine: repeat to safe BP
Find and Rx any infection Phenoxybenzamine: exchange for phentolamine
Longer t
2nd -block
HONK Only after -blockade
Avoids unopposed -adrenergic stimulation
Rehydrate
c 0.9% NS over 48h
Surgery
May need ~9L
Electively after 406wks to allow full -blockade
Wait 1h before starting insulin and volume expansion
It may not be needed Phenoxybenzamine d until significant postural
Start low to avoid rapid changes in osmolality hypotension.
e.g. 1-3u/hr
LMWH

Hypoglycaemia
Alert and Orientated: Oral Carb
Rapid acting: lucozade
Long acting: toast, sandwich

Drowsy / confused but swallow intact: Buccal Carb


Hypostop / Glucogel
Consider IV access

Unconscious or Concerned re Swallow: IV dextrose


50ml 50% or 100ml 20% glucose

Deteriorating / refractory / insulin-induced / no access


1mg glucagon IM/SC
Wont work in drunks + short duration of effect (20min)
Insulin release may rebound hypoglycaemia

Alasdair Scott, 2012 83


Metabolic
Hyperkalaemia Acute Poisoning
ECG Features (in order) Poison Rx
Peaked T waves Aspirin Alkalinise urine: NaHCO3 + KCl
Flattened P waves Benzodiazepine Flumazenil
PR interval -B Atropine, glucagon
Widened QRS Cyanide Dicobalt edentate
Sine-wave pattern VF CO Hyperbaric O2
Digoxin Digibind
Mx Ethylene Glycol Ethanol
Calcium gluconate 10ml 10% Heparin Protamine
50ml 50% glucose + 10u insulin (Actrapid) Iron Desferrioxamine
Salbutamol 5mg nebulizer Lead Sodium calcium edentate
Calcium resonium 15g PO or 30g PR Methanol Ethanol
Haemofiltration (usually needed if anuric) Opiates Naloxone
Organophosphates Atropine + pralidoxime
Paracetamol NAC
Hypokalaemia TCA NaHCO3 + O2
Warfarin: major bleed PCC + Vit K IV
ECG Features
Result from delayed ventricular repolarisation
Flattened / inverted T waves
Prominent U waves (after T waves)
ST depression
Long PR interval
Long QT interval

Mild: K >2.5
Oral K supplements
80mmol/d

Severe: K <2.5 and/or dangerous symptoms


IV K cautiously
10mmol/h (Max 20mmol/h)
Best to give centrally (burning sensation)
Max central conc: 60mM
Max peripheral conc: 40mM

Hypercalcaemia
Rehydrate
1L 0.9% NS / 4h
Monitor pts. hydration state

Frusemide
Only start once pt. is volume replete
Calciuric + makes room for more fluids

Bisphosphonates
Ca bisphosphonate cant be resorbed by osteoclasts
Used to prevent recurrence
Can obscure Dx as Ca, PO4 and PTH
E.g. Pamidronate, Zoledronate (IV)

Alasdair Scott, 2012 84


Revision
Contents
Prescribing Scenarios ................................................................................................................................................................. 86
Prescriptions for Comment .......................................................................................................................................................... 88
Useful Drugs to Know .................................................................................................................................................................. 90

Alasdair Scott, 2012 85


Prescribing Scenarios
Scenario 1 Scenario 4
68 year old man with T2DM and hypertension is brought to the A 59-year-old woman presents with a several year history of
A&E department with AF which started less than 2 hours increasing shortness of breath and a productive cough. She
earlier. His ventricular rate is about 130/min. DC cardioversion has a smoking history of at least 30 pack years but no other
is unsuccessful and he remains in hospital. He says he would medical history. On examination of the chest there is
prefer not to have a further attempt at the procedure. BP is widespread wheeze and coarse crackles cleared by coughing,
consistently higher that 150/90 during the daytime. but the sputum is very viscous though clear. Spirometry
Electrolytes, creatinine, and thyroid function are normal. confirms a mixture of obstructive and restrictive abnormalities.
Fasting blood glucose is 7.9, fasting total cholesterol is 6.6,
HDL cholesterol is 0.96 and TGs 3.4 Answer
Formoterol inhaler 12 micrograms x2 daily + extra
Answer doses for symptom relief (maximum total dose 48
Metformin 500mg x1 daily c breakfast micrograms daily)
Simvastatin 20mg x1 nocte Carbocysteine 750mg x3 daily
Verapamil 80mg x3 daily
Lisinopril 10mg x1 daily
Warfarin 5mg x1 daily Scenario 5
Check INR on day 5 and adjust dose according
to Tait Regimen, aiming for an INR of 2.5. 73 year old woman is admitted with congestive heart failure.
She has pitting oedema to the umbilicus and is almost
immobile. Her BMI is 28 and BP is 121/65. On admission she
Scenario 2 is found to have a UTI, bacteriology results are awaited. Other
investigations show creatinine 133, fasting Glucose 5.1, total
72 year old woman is admitted with a fractured neck of femur cholesterol 4.8, HDL cholesterol 1.1. CXR confirms pulmonary
following a fall. She has successful surgery but despite congestion.
appropriate prophylaxis develops a DVT in one calf. While she
is being treated for this, she undergoes bone densitometry Answer
which reveals vertebral and femoral osteoporosis. She also Oxygen 4L via nasal cannula: aim for SpO2 94-98%
has a lower UTI. Frusemide 40 mg IV x1 daily
Lisinopril 2.5mg x1 daily
Answer Bisoprolol 1.25mg x1 daily
Paracetamol 1g x4 daily Simvastatin 20mg x1 nocte
Codeine phosphate 30mg every 4 hours as required Aspirin 75mg x1 daily
Maximum daily dose 240mg Cephalexin 250mg x4 daily for 7 days
Enoxaparin 1.5mg/kg x1 daily SC until INR = 2.5 Enoxaparin 40mg x1 daily SC until mobile
Warfarin 5mg x1 daily
Check INR on day 5 and adjust dose according
to Tait regimen, aiming for an INR of 2.5.
Continue warfarin for 6 weeks 3 months.
Alendronate 70mg x1 weekly
Trimethoprim 200mg x2 daily for 7d

Scenario 3
A 62-year-old man is admitted with a 3-day-history of
increasing shortness of breath. On examination his
temperature is 37.7 C, he has right lower lobe consolidation
and also pitting oedema to his knees and JVP +6 cm. He is in
sinus rhythm at 88 bpm and his BP is 133/76 mm Hg and
oxygen saturation is 92%.

Answer
Oxygen 4L via nasal cannula: aim for SpO2 94-98%
Amoxicillin 500mg x3 daily for 7d
Clarithromycin 500mg x2 daily for 7d
Frusemide 40mg x1 daily

Alasdair Scott, 2012 86


Scenario 6
A 73-year-old woman is admitted with a fractured neck of
femur following a fall. She had been lying on the floor of her
flat for at least 24 hours and was found to have a calf deep
vein thrombosis and a lower urinary tract infection. There were
no other abnormalities found on admission and she was not
known to have any other illnesses before her accident. She
has moderately severe pain and surgery is planned but for the
moment she is not nil by mouth.

Answer
Paracetamol 1g x4 daily
Tramadol 50mg every 4 hours as required
Maximum daily dose 300mg
Enoxaparin 1.5mg/kg x1 daily SC
No warfarin before surgery
Trimethoprim 200mg x2 daily for 7d

Scenario 7
A 62-year-old man with type 2 diabetes and hypertension is
admitted with a 3-day history of increasing cough and
shortness of breath, and a temperature of up to 38.2C. His
BMI is approximately 31kg/m2. On examination there are
signs of right lower lobe consolidation. Blood pressure varies
between 144/91 and 163/103 mm Hg. Electrolytes and
creatinine are within the normal range. Random blood
glucose is 14. 3 mmol/L, fasting total cholesterol is 6.1
mmol/L, HDL cholesterol 0.96 mmol/L and triglycerides 3.4
mmol/l.

Answer
Oxygen 4L via nasal cannula: aim for SpO2 94-98%
Amoxicillin 500mg x3 daily for 7d
Clarithromycin 500mg x2 daily for 7d
Lisinopril 2.5mg x1 daily
Nifedipine MR 20mg x1 daily
Metformin 500mg x1 daily c breakfast
Simvastatin 20mg x1 nocte

Alasdair Scott, 2012 87


Prescriptions for Comment
Prescription 1 Prescription 2
69 year old man has COPD with heart failure. He also has 68 year old woman has PD for at least 5 years. She suffers a
BPH with obstructive symptoms. He is admitted with an TIA affecting her speech and the right hand side of her face
infective exacerbation of the COPD. lasting about 6 hours. BP is 158/62 seated and 132/54
standing, in sinus rhythm. Total cholesterol is 7.8, HDL 1.2.
Bendroflumethazide 5mg OD She is not diabetic and her renal and liver function are normal.
Bisoprolol 5mg OD There is 60% stenosis of the left internal carotid and 15% of
Ciprofloxacin 500 mg BD the right.
Doxazosin 8mg OD
Enalapril 5mg OD Aspirin 75mg OD
Ipratropium Bromide 40 ug QDS inhaled Atenolol 50mg OD
Clopidogrel 75mg OD
Comments Co-careldopa 250/25 BD
Bendroflumethazide Pravastatin 20mg OD
Not the preferred diuretic in heart failure. Switch
to frusemide. Comments
Bisoprolol Aspirin and Clopidogrel
High dose for heart failure. Titrate up slowly from TIA occurred despite aspirin and clopidogrel. The
1.25mg OD. MATCH trial found that the combination of aspirin
COPD was considered a relative contraindication and clopidogrel was not beneficial in
in COPD, but this is now controversial. A recent neurovascular disease and resulted in increased
trial in the BMJ suggested that cardioselective - bleeding risk. Stop the clopidogrel and add
blockers may be beneficial. dipyridamole MR. The NASCET and ECST trials
Ciprofloxacin demonstrated benefit of carotid endarterectomy
Not the preferred choice for acute exacerbation in symptomatic patients with 50-69% stenosis if
of COPD. Switch to amoxicillin. the operative risk is low. This option should be
Doxazosin considered.
High dose for treatment of BPH. Either reduce to Atenolol
2-4mg x1 daily or switch to tamsulosin which may Not the first line drug for the treatment of
be more effective. hypertension in patients over 55 years. In
Enalapril addition she demonstrates a significant postural
Dose is low. Gradually increase to 10-20mg BD if drop. Switch her antihypertensive to nifedipine
tolerated. MR 20mg x1 daily.
Ipratropium bromide Pravastatin
May exacerbate BPH. Switch to formoterol Cholesterol is high despite treatment c
inhaler. pravastatin. Pravastatin has a relatively low
potency and the dose is low. Switch to
simvastatin 20mg x1 nocte.
Co-careldopa
Dose is expressed incorrectly: dose of carbidopa
should be given first: i.e. 25/250 x2 daily.
Dose is quite high and not well spaced out (L-
DOPA has a short t). In addition she has
postural hypotension. Change regimen to 25/100
x3-4 daily.

Alasdair Scott, 2012 88


Prescription 3 Prescription 4
A 65-year-old man has COPD and permanent atrial fibrillation A 72-year-old woman has Parkinsons disease and develops
with mild heart failure. He is admitted with an infective nausea on initiation of treatment. She has had isolated
exacerbation of the COPD. systolic hypertension for at least 5 years. A few weeks later
she develops distressing visual hallucinations. Her blood
Verapamil modified release 120 mg x1 daily pressure is 117/62 mm hg seated and 98/55 mm Hg standing.
Bisoprolol 5 mg x1 daily
Clarithromycin 500 mg x 2 daily Co-careldopa 250/25 1 tablet x 4 daily
Warfarin 4 mg x 1 daily (INR 2.8) Metoclopramide 10 mg x 3 daily
Lisinopril 20 mg x1 daily Prochlorperazine 5 mg x 1 at bedtime
Furosemide 40 mg x1 daily Irbesartan 150 mg x 1 daily

Answer Answer
Bisoprolol Co-careldopa
High dose for heart failure. Titrate up slowly from Dose is expressed incorrectly: dose of carbidopa
1.25mg OD. should be given first: i.e. 25/250 x2 daily.
COPD was considered a relative contraindication Dose is quite high. In addition she is experiencing
in COPD, but this is now controversial. A recent hallucinations and has postural hypotension.
trial in the BMJ suggested that cardioselective - Reduce dose: e.g. 25/100 x3-4 daily.
blockers may be beneficial. Irbesartan
-B + verapamil contraindicated Dose is high and isnt first line antihypertensive in
Clarithromycin + Warfarin this age. May not even be necessary at all given
Clarithromycin is a Cyp450 inhibitor. The dose of her blood pressure.
warfarin may need to be reduced Metoclopramide and prochlorperazine
Verapamil + Furosemide + Lisinopril Unnecessary to be on two anti-emetics and both
Multiple drugs with antihypertensive effects may are dopamine antagonists that cross the BBB
result in hypotension. and will worsen her parkinsonian symptoms. Stop
There is no specific COPD therapy. them both and use domperidone to control her
nausea.
If her psychiatric symptoms persist after the dose
st
of co-careldopa has been reduced, 1 line
treatment would be an atypical anti-psychotic,
such as quetiapine.

Alasdair Scott, 2012 89


Useful Drugs to Know

Drug Dose
Amoxicillin 500mg TDS PO
Clarithromycin 500mg BD PO
Trimethoprim 200mg BD PO
Co-amoxiclav 1.2g TDS IV
Simvastatin 20mg OD nocte PO
Nifedipine MR 20mg OD PO
Lisinopril HTN: 10mg OD PO
HF: 2.5mg OD PO
Bisoprolol HF: 1.25mg OD PO
Paracetamol 1g QDS PO
Codeine Phosphate 30mg every 4h PRN PO
Max 240mg daily
Tramadol 50mg every 4h PRN PO
Max 300mg daily
Enoxaparin Treatment: 1.5mg/kg/24h SC
Prophylaxis: 40mg OD SC

Alasdair Scott, 2012 90

Potrebbero piacerti anche